Vous êtes sur la page 1sur 103

ORBITAL MECHANICS

Conic Sections

Orbital Elements

Types of Orbits

Newton's Laws of Motion and Universal Gravitation

Uniform Circular Motion

Motions of Planets and Satellites

Launch of a Space Vehicle

Position in an Elliptical Orbit

Orbit Perturbations

Orbit Maneuvers

The Hyperbolic Orbit

Orbital mechanics, also called flight mechanics, is the study of the motions of artificial satellites and space vehicles moving
under the influence of forces such as gravity, atmospheric drag, thrust, etc. Orbital mechanics is a modern offshoot of celestial
mechanics which is the study of the motions of natural celestial bodies such as the moon and planets. The root of orbital
mechanics can be traced back to the 17th century when mathematician Isaac Newton (1642-1727) put forward his laws of
motion and formulated his law of universal gravitation. The engineering applications of orbital mechanics include ascent
trajectories, reentry and landing, rendezvous computations, and lunar and interplanetary trajectories.

Conic Sections
A conic section, or just conic, is a curve formed by passing a plane through a right circular cone. As shown in Figure 4.1, the
angular orientation of the plane relative to the cone determines whether the conic section is a circle, ellipse, parabola, or
hyperbola. The circle and the ellipse arise when the intersection of cone and plane is a bounded curve. The circle is a special
case of the ellipse in which the plane is perpendicular to the axis of the cone. If the plane is parallel to a generator line of the
cone, the conic is called a parabola. Finally, if the intersection is an unbounded curve and the plane is not parallel to a generator
line of the cone, the figure is a hyperbola. In the latter case the plane will intersect both halves of the cone, producing two
separate curves.
We can define all conic sections in terms of the eccentricity. The type of conic section is also related to the semi-major axis and
the energy. The table below shows the relationships between eccentricity, semi-major axis, and energy and the type of conic
section.
Conic Section

Eccentricity, e

Semi-major axis

Energy

Circle

= radius

<0

Ellipse

0<e<1

>0

<0

Parabola

infinity

Hyperbola

>1

<0

>0

Satellite orbits can be any of the four conic sections. This page deals mostly with elliptical orbits, though we conclude with an
examination of the hyperbolic orbit.

Orbital Elements
To mathematically describe an orbit one must define six
quantities, called orbital elements. They are

Semi-Major Axis, a

Eccentricity, e

Inclination, i

Argument of Periapsis,

Time of Periapsis Passage, T

Longitude of Ascending Node,

An orbiting satellite follows an oval shaped path known as


an ellipse with the body being orbited, called the primary,
located at one of two points called foci. An ellipse is
defined to be a curve with the following property: for each point on an ellipse, the sum of its distances from two fixed points,
called foci, is constant (see Figure 4.2). The longest and shortest lines that can be drawn through the center of an ellipse are
called the major axis and minor axis, respectively. The semi-major axis is one-half of the major axis and represents a satellite's
mean distance from its primary. Eccentricity is the distance between the foci divided by the length of the major axis and is a
number between zero and one. An eccentricity of zero indicates a circle.

Inclination is the angular distance between a satellite's orbital plane and the equator of
its primary (or the ecliptic plane in the case of heliocentric, or sun centered, orbits). An
inclination of zero degrees indicates an orbit about the primary's equator in the same
direction as the primary's rotation, a direction called prograde (or direct). An inclination
of 90 degrees indicates a polar orbit. An inclination of 180 degrees indicates a
retrograde equatorial orbit. A retrograde orbit is one in which a satellite moves in a
direction opposite to the rotation of its primary.
Periapsis is the point in an orbit closest to the primary. The opposite of periapsis, the
farthest point in an orbit, is called apoapsis. Periapsis and apoapsis are usually modified
to apply to the body being orbited, such as perihelion and aphelion for the Sun, perigee
and apogee for Earth, perijove and apojove for Jupiter, perilune and apolune for the
Moon, etc. The argument of periapsis is the angular distance between the ascending
node and the point of periapsis (see Figure 4.3). The time of periapsis passage is the
time in which a satellite moves through its point of periapsis.
Nodes are the points where an orbit crosses a plane, such as a satellite crossing the
Earth's equatorial plane. If the satellite crosses the plane going from south to north, the node is the ascending node; if moving
from north to south, it is the descending node. The longitude of the ascending node is the node's celestial longitude. Celestial
longitude is analogous to longitude on Earth and is measured in degrees counter-clockwise from zero with zero longitude being
in the direction of the vernal equinox.

In general, three observations of an object in orbit are required to calculate the six orbital elements. Two other quantities often
used to describe orbits are period and true anomaly. Period, P, is the length of time required for a satellite to complete one
orbit. True anomaly, , is the angular distance of a point in an orbit past the point of periapsis, measured in degrees.

Types Of Orbits
For a spacecraft to achieve Earth orbit, it must be launched to an elevation above the Earth's atmosphere and accelerated to
orbital velocity. The most energy efficient orbit, that is one that requires the least amount of propellant, is a direct low
inclination orbit. To achieve such an orbit, a spacecraft is launched in an eastward direction from a site near the Earth's equator.
The advantage being that the rotational speed of the Earth contributes to the spacecraft's final orbital speed. At the United
States' launch site in Cape Canaveral (28.5 degrees north latitude) a due east launch results in a "free ride" of 1,471 km/h (914
mph). Launching a spacecraft in a direction other than east, or from a site far from the equator, results in an orbit of higher
inclination. High inclination orbits are less able to take advantage of the initial speed provided by the Earth's rotation, thus the
launch vehicle must provide a greater part, or all, of the energy required to attain orbital velocity. Although high inclination
orbits are less energy efficient, they do have advantages over equatorial orbits for certain applications. Below we describe
several types of orbits and the advantages of each:
Geosynchronous orbits (GEO) are circular orbits around the Earth having a period of 24 hours. A geosynchronous orbit with
an inclination of zero degrees is called a geostationary orbit. A spacecraft in a geostationary orbit appears to hang motionless

above one position on the Earth's equator. For this reason, they are ideal for some types of communication and meteorological
satellites. A spacecraft in an inclined geosynchronous orbit will appear to follow a regular figure-8 pattern in the sky once every
orbit. To attain geosynchronous orbit, a spacecraft is first launched into an elliptical orbit with an apogee of 35,786 km (22,236
miles) called a geosynchronous transfer orbit (GTO). The orbit is then circularized by firing the spacecraft's engine at apogee.
Polar orbits (PO) are orbits with an inclination of 90 degrees. Polar orbits are useful for satellites that carry out mapping
and/or surveillance operations because as the planet rotates the spacecraft has access to virtually every point on the planet's
surface.
Walking orbits: An orbiting satellite is subjected to a great many gravitational influences. First, planets are not perfectly
spherical and they have slightly uneven mass distribution. These fluctuations have an effect on a spacecraft's trajectory. Also,
the sun, moon, and planets contribute a gravitational influence on an orbiting satellite. With proper planning it is possible to
design an orbit which takes advantage of these influences to induce a precession in the satellite's orbital plane. The resulting
orbit is called a walking orbit, or precessing orbit.
Sun synchronous orbits (SSO) are walking orbits whose orbital plane precesses with the same period as the planet's solar
orbit period. In such an orbit, a satellite crosses periapsis at about the same local time every orbit. This is useful if a satellite is
carrying instruments which depend on a certain angle of solar illumination on the planet's surface. In order to maintain an exact
synchronous timing, it may be necessary to conduct occasional propulsive maneuvers to adjust the orbit.
Molniya orbits are highly eccentric Earth orbits with periods of approximately 12 hours (2 revolutions per day). The orbital
inclination is chosen so the rate of change of perigee is zero, thus both apogee and perigee can be maintained over fixed
latitudes. This condition occurs at inclinations of 63.4 degrees and 116.6 degrees. For these orbits the argument of perigee is
typically placed in the southern hemisphere, so the satellite remains above the northern hemisphere near apogee for
approximately 11 hours per orbit. This orientation can provide good ground coverage at high northern latitudes.
Hohmann transfer orbits are interplanetary trajectories whose advantage is that they consume the least possible amount of
propellant. A Hohmann transfer orbit to an outer planet, such as Mars, is achieved by launching a spacecraft and accelerating it
in the direction of Earth's revolution around the sun until it breaks free of the Earth's gravity and reaches a velocity which places
it in a sun orbit with an aphelion equal to the orbit of the outer planet. Upon reaching its destination, the spacecraft must
decelerate so that the planet's gravity can capture it into a planetary orbit.
To send a spacecraft to an inner planet, such as Venus, the spacecraft is launched and accelerated in the direction opposite of
Earth's revolution around the sun (i.e. decelerated) until it achieves a sun orbit with a perihelion equal to the orbit of the inner
planet. It should be noted that the spacecraft continues to move in the same direction as Earth, only more slowly.

To reach a planet requires that the spacecraft be inserted into an interplanetary trajectory at the correct time so that the
spacecraft arrives at the planet's orbit when the planet will be at the point where the spacecraft will intercept it. This task is
comparable to a quarterback "leading" his receiver so that the football and receiver arrive at the same point at the same time.
The interval of time in which a spacecraft must be launched in order to complete its mission is called a launch window.

Newton's Laws of Motion and Universal Gravitation


Newton's laws of motion describe the relationship between the motion of a particle and the forces acting on it.
The first law states that if no forces are acting, a body at rest will remain at rest, and a body in motion will remain in motion in
a straight line. Thus, if no forces are acting, the velocity (both magnitude and direction) will remain constant.
The second law tells us that if a force is applied there will be a change in velocity, i.e. an acceleration, proportional to the
magnitude of the force and in the direction in which the force is applied. This law may be summarized by the equation

where F is the force, m is the mass of the particle, and a is the acceleration.
The third law states that if body 1 exerts a force on body 2, then body 2 will exert a force of equal strength, but opposite in
direction, on body 1. This law is commonly stated, "for every action there is an equal and opposite reaction".
In his law of universal gravitation, Newton states that two particles having masses m1 and m2 and separated by a distance r are
attracted to each other with equal and opposite forces directed along the line joining the particles. The common magnitude F of
the two forces is

where G is an universal constant, called the constant of gravitation, and has the value 6.67259x10-11 N-m2/kg2 (3.4389x10-8 lbft2/slug2).
Let's now look at the force that the Earth exerts on an object. If the object has a mass m, and the Earth has mass M, and the
object's distance from the center of the Earth is r, then the force that the Earth exerts on the object is GmM /r2 . If we drop the

object, the Earth's gravity will cause it to accelerate toward the center of the Earth. By Newton's second law (F = ma), this
acceleration g must equal (GmM /r2)/m, or

At the surface of the Earth this acceleration has the valve 9.80665 m/s2 (32.174 ft/s2).
Many of the upcoming computations will be somewhat simplified if we express the product GM as a constant, which for Earth
has the value 3.986005x1014 m3/s2 (1.408x1016 ft3/s2). The product GM is often represented by the Greek letter .

BASIC CONSTANTS
Mathematical Constants
3.141592653589793
e

2.718281828459045
Physical Constants

Speed of light (c)

299,792,458

m/s

Constant of gravitation (G)

6.67259x10-11

Nm2/kg2

Universal gas constant (R)

8,314.4621

J/kmol-K

Stefan-Boltzmann constant ()

5.670373x10-8

W/m2-K4

Acceleration of gravity (g)

9.80665

m/s2

Standard atmosphere, sea level

101,325

Pa

Astronomical Constants
Astronomical unit (AU)

149,597,870

km

Light year (ly)

9.460530x1012

km

Parsec (pc)

3.261633

ly

Sidereal year

365.256366

days

Mass of Sun

1.9891x1030

kg

Radius of Sun

696,000

km

Mass of Earth

5.9737x1024

kg

Equatorial radius of Earth

6,378.137

km

Earth oblateness

1/298.257

Obliquity of the ecliptic, epoch 2000

23.4392911

degrees

Mean lunar distance

384,403

km

Radius of Moon

1,738

km

Mass of Moon

7.348x1022

kg

Luminosity of Sun

3.839x1026

Solar constant, at 1 AU

1,366

W/m2

Solar maxima

1990 + 11n

(date)

Spaceflight Constants
GM (Sun)

1.32712438x1020

m3/s2

GM (Earth)

3.986005x1014

m3/s2

GM (Moon)

4.902794x1012

m3/s2

GM (Mars)

4.282831x1013

m3/s2

J2 (Earth)

0.00108263

J2 (Moon)

0.0002027

J2 (Mars)

0.00196045

GM
(1015 m3/s2)

Semimajor Axis
(106 km)

0.02203

57.91

0.3249

108.21

0.3986

149.60

0.04283

227.92

126.686

778.57

37.931

1,433.53

5.794

2,872.46

6.835

4,495.06

0.00083

5,869.66

WEIGHTS & MEASURES

Two systems of weights and measures coexist in the United States today: the U.S. Customary System and the International
System of Units (SI, after the initials Systeme International). The SI System, commonly identified with the metric system, is
actually a more complete and coherent version of it. Throughout U.S. history, the Customary System (inherited from, but now
different from, the British Imperial System) has been generally used.
The use of the SI System has slowly and steadily increased in the United States, particularly in the scientific community;
however, the general public still uses, almost exclusively, the U.S. Customary System. Because this Web site has a large
international audience, all calculations and example problems make exclusive use of SI units, however the formulae provided
will work in either system of units. Whenever a basic constant is given, its U.S. Customary equivalent is also given in
parentheses.
In the SI System, the basic units are the units of length, mass, and time, and are called respectively, the meter (m), the
kilogram (kg), and the second (s). The unit of force, called the Newton (N), is a derived unit and is defined as the force that
gives an acceleration of 1 m/s2 to a mass of 1 kg.
In the U.S. Customary System the base units are the units of length, force, and time, and are called respectively, the foot (ft),
the pound (lb), and the second (s). The unit of mass, called the slug, is a derived unit and is defined as the mass that receives
an acceleration of 1 ft/s2 when a force of 1 lb is applied to it.
When working in the U.S. Customary System, it is commonplace to express "mass" in pounds; however, when doing so, it is
necessary to recognize that we are actually expressing "weight", which is the measure of the gravitational force on a body.
When used in this way, the weight is that of a mass when it is subjected to an acceleration of one g. In the study of dynamics,
where forces, masses, and accelerations are involved, it is important that we express the mass m in slugs of a body of which the
weight W has been given in pounds. That is, m=W/g, where g is approximately equal to 32.174 ft/s2.
It is sometimes common practice for mathematical equations to be expressed in such a way that the variable for mass is
entered in the unit of pounds. These equations include a term that converts mass from pounds into its proper unit of slugs.
When using these equations, one must proceed with caution because they are only valid when using U.S. Customary units.
When using SI units, it is necessary to use a different equation that does not contain the conversion factor. To avoid this
problem, all equations used in this Web site require that mass be expressed in the unit of slugs in the U.S. Customary System,
and kilograms in the SI System.

Uniform Circular Motion

In the simple case of free fall, a particle accelerates toward the center of the Earth while moving in a straight line. The velocity
of the particle changes in magnitude, but not in direction. In the case of uniform circular motion a particle moves in a circle with
constant speed. The velocity of the particle changes continuously in direction, but not in magnitude. From Newton's laws we see
that since the direction of the velocity is changing, there is an acceleration. This acceleration, called centripetal acceleration is
directed inward toward the center of the circle and is given by

where v is the speed of the particle and r is the radius of the circle. Every accelerating particle must have a force acting on it,
defined by Newton's second law (F = ma). Thus, a particle undergoing uniform circular motion is under the influence of a force,
called centripetal force, whose magnitude is given by

The direction of F at any instant must be in the direction of a at the same instant, that is radially inward.
A satellite in orbit is acted on only by the forces of gravity. The inward acceleration which causes the satellite to move in a
circular orbit is the gravitational acceleration caused by the body around which the satellite orbits. Hence, the satellite's
centripetal acceleration is g, that is g = v2/r. From Newton's law of universal gravitation we know that g = GM /r2. Therefore, by
setting these equations equal to one another we find that, for a circular orbit,

Motions of Planets and Satellites

Through a lifelong study of the motions of bodies in the solar system, Johannes Kepler (1571-1630) was able to derive three
basic laws known as Kepler's laws of planetary motion. Using the data compiled by his mentor Tycho Brahe (1546-1601), Kepler
found the following regularities after years of laborious calculations:
1. All planets move in elliptical orbits with the sun at one focus.
2. A line joining any planet to the sun sweeps out equal areas in equal times.
3. The square of the period of any planet about the sun is proportional to the cube of the planet's mean distance from the sun.
These laws can be deduced from Newton's laws of motion and law of universal gravitation. Indeed, Newton used Kepler's work
as basic information in the formulation of his gravitational theory.
As Kepler pointed out, all planets move in elliptical orbits, however, we can learn much about planetary motion by considering
the special case of circular orbits. We shall neglect the forces between planets, considering only a planet's interaction with the
sun. These considerations apply equally well to the motion of a satellite about a planet.
Let's examine the case of two bodies of masses M and m moving in circular orbits under the influence of each other's
gravitational attraction. The center of mass of this system of two bodies lies along the line joining them at a point C such that
mr = MR. The large body of mass M moves in an orbit of constant radius R and the small body of mass m in an orbit of constant
radius r, both having the same angular velocity . For this to happen, the gravitational force acting on each body must provide
the necessary centripetal acceleration. Since these gravitational forces are a simple action-reaction pair, the centripetal forces
must be equal but opposite in direction. That is, m 2r must equal M 2R. The specific requirement, then, is that the gravitational
force acting on either body must equal the centripetal force needed to keep it moving in its
circular orbit, that is

If one body has a much greater mass than the other, as is the case of the sun and a planet or
the Earth and a satellite, its distance from the center of mass is much smaller than that of the
other body. If we assume that m is negligible compared to M, then R is negligible compared to
r. Thus, equation (4.7) then becomes

If we express the angular velocity in terms of the period of revolution,

= 2 /P, we obtain

where P is the period of revolution. This is a basic equation of planetary and satellite motion. It also holds for elliptical orbits if
we define r to be the semi-major axis (a) of the orbit.
A significant consequence of this equation is that it predicts Kepler's third law of planetary motion, that is P2~r3.

In celestial mechanics where we are dealing with planetary or stellar sized bodies, it is often the case that the mass of the secondary body is
significant in relation to the mass of the primary, as with the Moon and Earth. In this case the size of the secondary cannot be ignored. The distance R
is no longer negligible compared to r and, therefore, must be carried through the derivation. Equation (4.9) becomes

More commonly the equation is written in the equivalent form

where a is the semi-major axis. The semi-major axis used in astronomy is always the primary-to-secondary distance, or the geocentric semi-major
axis. For example, the Moon's mean geocentric distance from Earth (a) is 384,403 kilometers. On the other hand, the Moon's distance from the
barycenter (r) is 379,732 km, with Earth's counter-orbit (R) taking up the difference of 4,671 km.

Kepler's second law of planetary motion must, of course, hold true for circular orbits. In such orbits both and r are constant so
that equal areas are swept out in equal times by the line joining a planet and the sun. For elliptical orbits, however, both and r
will vary with time. Let's now consider this case.
Figure 4.5 shows a particle revolving around C along some arbitrary path. The area swept out by the radius vector in a short
time interval t is shown shaded. This area, neglecting the small triangular region at the end, is one-half the base times the
height or approximately r(r
t)/2. This expression becomes more exact as t approaches zero, i.e. the small triangle goes to
zero more rapidly than the large one. The rate at which area is being swept out
instantaneously is therefore

For any given body moving under the influence of a central force, the value

r2 is constant.

Let's now consider two points P1 and P2 in an orbit with radii r1 and r2, and velocities v1 and
v2. Since the velocity is always tangent to the path, it can be seen that if is the angle
between r and v, then

where vsin

is the transverse component of v. Multiplying through by r, we have

or, for two points P1 and P2 on the orbital path

Note that at periapsis and apoapsis,


points we get

= 90 degrees. Thus, letting P1 and P2 be these two

Let's now look at the energy of the above particle at points P1 and P2. Conservation of energy
states that the sum of the kinetic energy and the potential energy of a particle remains
constant. The kinetic energy T of a particle is given by mv2/2 while the potential energy of
gravity V is calculated by the equation -GMm/r. Applying conservation of energy we have

From equations (4.14) and (4.15) we obtain

Rearranging terms we get

The eccentricity e of an orbit is given by

If the semi-major axis a and the eccentricity e of an orbit are known, then the periapsis and apoapsis distances can be
calculated by

Launch of a Space Vehicle

The launch of a satellite or space vehicle consists of a period of powered flight during which the vehicle is lifted above the
Earth's atmosphere and accelerated to orbital velocity by a rocket, or launch vehicle. Powered flight concludes at burnout of the
rocket's last stage at which time the vehicle begins its free flight. During free flight the space vehicle is assumed to be subjected
only to the gravitational pull of the Earth. If the vehicle moves far from the Earth, its trajectory may be affected by the
gravitational influence of the sun, moon, or another planet.
A space vehicle's orbit may be determined from the position and the velocity of the
vehicle at the beginning of its free flight. A vehicle's position and velocity can be
described by the variables r, v, and , where r is the vehicle's distance from the center
of the Earth, v is its velocity, and is the angle between the position and the velocity
vectors, called the zenith angle (see Figure 4.7). If we let r1, v1, and 1 be the initial
(launch) values of r, v, and , then we may consider these as given quantities. If we let
point P2 represent the perigee, then equation (4.13) becomes

Substituting equation (4.23) into (4.15), we can obtain an equation for the perigee
radius Rp.

Multiplying through by -Rp2/(r12v12) and rearranging, we get

Note that this is a simple quadratic equation in the ratio (Rp/r1) and that 2GM /(r1 v12) is a nondimensional parameter of the
orbit.

Solving for (Rp/r1) gives

Like any quadratic, the above equation yields two answers. The smaller of the two answers corresponds to Rp, the periapsis
radius. The other root corresponds to the apoapsis radius, Ra.
Please note that in practice spacecraft launches are usually terminated at either perigee or apogee, i.e.
results in the minimum use of propellant.

= 90. This condition

Equation (4.26) gives the values of Rp and Ra from which the eccentricity of the orbit can be calculated, however, it may be
simpler to calculate the eccentricity e directly from the equation

To pin down a satellite's orbit in space, we need to know the angle


point. This angle is given by

, the true anomaly, from the periapsis point to the launch

In most calculations, the complement of the zenith angle is used, denoted by . This angle is called the flight-path angle, and is
positive when the velocity vector is directed away from the primary as shown in Figure 4.8. When flight-path angle is used,
equations (4.26) through (4.28) are rewritten as follows:

The semi-major axis is, of course, equal to (Rp+Ra)/2, though it may be easier to calculate it directly as follows:

If e is solved for directly using equation (4.27) or (4.30), and a is solved for using equation (4.32), Rp and Ra can be solved for
simply using equations (4.21) and (4.22).
Orbit Tilt, Rotation and Orientation
Above we determined the size and shape of the orbit, but to determine the orientation of the orbit in space, we must know the
latitude and longitude and the heading of the space vehicle at burnout.

Figure 4.9 above illustrates the location of a space vehicle at engine burnout, or orbit insertion. is the azimuth heading
measured in degrees clockwise from north, is the geocentric latitude (or declination) of the burnout point,
is the angular
distance between the ascending node and the burnout point measured in the equatorial plane, and is the angular distance
between the ascending node and the burnout point measured in the orbital plane. 1 and 2 are the geographical longitudes of
the ascending node and the burnout point at the instant of engine burnout. Figure 4.10 pictures the orbital elements, where i is
the inclination, is the longitude at the ascending node, is the argument of periapsis, and is the true anomaly.
If

, and

are given, the other values can be calculated from the following relationships:

In equation (4.36), the value of is found using equation (4.28) or (4.31). If is positive, periapsis is west of the burnout point
(as shown in Figure 4.10); if is negative, periapsis is east of the burnout point.
The longitude of the ascending node, , is measured in celestial longitude, while 1 is geographical longitude. The celestial
longitude of the ascending node is equal to the local apparent sidereal time, in degrees, at longitude 1 at the time of engine
burnout. Sidereal time is defined as the hour angle of the vernal equinox at a specific locality and time; it has the same value as
the right ascension of any celestial body that is crossing the local meridian at that same instant. At the moment when the vernal
equinox crosses the local meridian, the local apparent sidereal time is 00:00.

Geodetic Latitude, Geocentric Latitude, and Declination


Latitude is the angular distance of a point on Earth's surface north or south of Earth's equator, positive north and negative south. The geodetic latitude
(or geographical latitude), , is the angle defined by the intersection of the reference ellipsoid normal through the point of interest and the true
equatorial plane. The geocentric latitude, ', is the angle between the true equatorial plane and the radius vector to the point of intersection of the
reference ellipsoid and the reference ellipsoid normal passing through the point of interest. Declination, , is the angular distance of a celestial object
north or south of Earth's equator. It is the angle between the geocentric radius vector to the
object of interest and the true equatorial plane.
R is the magnitude of the reference ellipsoid's geocentric radius vector to the point of interest
on its surface, r is the magnitude of the geocentric radius vector to the celestial object of
interest, and the altitude h is the perpendicular distance from the reference ellipsoid to the
celestial object of interest. The value of R at the equator is a, and the value of R at the poles is
b. The ellipsoid's flattening, f, is the ratio of the equatorial-polar length difference to the
equatorial length. For Earth, a equals 6,378,137 meters, b equals 6,356,752 meters, and f equals
1/298.257.
When solving problems in orbital mechanics, the measurements of greatest usefulness are the
magnitude of the radius vector, r, and declination, , of the object of interest. However, we are
often given, or required to report, data in other forms. For instance, at the time of some specific
event, such as "orbit insertion", we may be given the spacecraft's altitude along with the
geodetic latitude and longitude of the sub-vehicle point. In such cases, it may be necessary to convert the given data to a form more suitable for our

calculations.
The relationship between geodetic and geocentric latitude is,

The radius of the reference ellipsoid is given by,

The length r can be solved from h, or h from r, using one of the following,

And declination is calculated using,

For spacecraft in low earth orbit, the difference between and ' is very small, typically not more than about 0.00001 degree. Even at the distance of
the Moon, the difference is not more than about 0.01 degree. Unless very high accuracy is needed, for operations near Earth we can assume '
and r R + h.

It is important to note that the value of h is not always measured as described and illustrated above. In some applications it is customary to express h
as the perpendicular distance from a reference sphere, rather than the reference ellipsoid. In this case, R is considered constant and is often assigned
the value of Earth's equatorial radius, hence h = r a. This is the method typically used when a spacecraft's orbit is expressed in a form such as "180
km 220 km". The example problems presented in this web site also assume this method of measurement.

Position in an Elliptical Orbit


Johannes Kepler was able to solve the problem of relating position in an orbit to the elapsed time, t-to, or conversely, how long it
takes to go from one point in an orbit to another. To solve this, Kepler introduced the quantity M, called the mean anomaly,
which is the fraction of an orbit period that has elapsed since perigee. The mean anomaly equals the true anomaly for a circular
orbit. By definition,

where Mo is the mean anomaly at time to and n is the mean motion, or the average angular velocity, determined from the semimajor axis of the orbit as follows:

This solution will give the average position and velocity, but satellite orbits are elliptical with a radius constantly varying in orbit.
Because the satellite's velocity depends on this varying radius, it changes as well. To resolve this problem we can define an
intermediate variable E, called the eccentric anomaly, for elliptical orbits, which is given by

where

is the true anomaly. Mean anomaly is a function of eccentric anomaly by the formula

For small eccentricities a good approximation of true anomaly can be obtained by the following formula (the error is of the order
e3):

The preceding five equations can be used to (1) find the time it takes to go from one position in an orbit to another, or (2) find
the position in an orbit after a specific period of time. When solving these equations it is important to work in radians rather
than degrees, where 2 radians equals 360 degrees.
At any time in its orbit, the magnitude of a spacecraft's position vector, i.e. its distance from the primary body, and its flightpath angle can be calculated from the following equations:

And the spacecraft's velocity is given by,

Orbit Perturbations
The orbital elements discussed at the beginning of this section provide an excellent reference for describing orbits, however
there are other forces acting on a satellite that perturb it away from the nominal orbit. These perturbations, or variations in the
orbital elements, can be classified based on how they affect the Keplerian elements. Secular variations represent a linear
variation in the element, short-period variations are periodic in the element with a period less than the orbital period, and longperiod variations are those with a period greater than the orbital period. Because secular variations have long-term effects on
orbit prediction (the orbital elements affected continue to increase or decrease), they will be discussed here for Earth-orbiting
satellites. Precise orbit determination requires that the periodic variations be included as well.

Third-Body Perturbations
The gravitational forces of the Sun and the Moon cause periodic variations in all of the orbital elements, but only the longitude of
the ascending node, argument of perigee, and mean anomaly experience secular variations. These secular variations arise from
a gyroscopic precession of the orbit about the ecliptic pole. The secular variation in mean anomaly is much smaller than the
mean motion and has little effect on the orbit, however the secular variations in longitude of the ascending node and argument
of perigee are important, especially for high-altitude orbits.
For nearly circular orbits the equations for the secular rates of change resulting from the Sun and Moon are
Longitude of the ascending node:

Argument of perigee:

where i is the orbit inclination, n is the number of orbit revolutions per day, and
and are in degrees per day. These equations
are only approximate; they neglect the variation caused by the changing orientation of the orbital plane with respect to both the
Moon's orbital plane and the ecliptic plane.

Click here for example problem #4.16


Perturbations due to Non-spherical Earth
When developing the two-body equations of motion, we assumed the Earth was a spherically symmetrical, homogeneous mass.
In fact, the Earth is neither homogeneous nor spherical. The most dominant features are a bulge at the equator, a slight pear
shape, and flattening at the poles. For a potential function of the Earth, we can find a satellite's acceleration by taking the

gradient of the potential function. The most widely used form of the geopotential function depends on latitude and geopotential
coefficients, Jn, called the zonal coefficients.
The potential generated by the non-spherical Earth causes periodic variations in all the orbital elements. The dominant effects,
however, are secular variations in longitude of the ascending node and argument of perigee because of the Earth's oblateness,
represented by the J2 term in the geopotential expansion. The rates of change of and due to J2 are

where n is the mean motion in degrees/day, J2 has the value 0.00108263, RE is the Earth's equatorial radius, a is the semimajor axis in kilometers, i is the inclination, e is the eccentricity, and and are in degrees/day. For satellites in GEO and
below, the J2 perturbations dominate; for satellites above GEO the Sun and Moon perturbations dominate.
Molniya orbits are designed so that the perturbations in argument of perigee are zero. This conditions occurs when the term 45sin2i is equal to zero or, that is, when the inclination is either 63.4 or 116.6 degrees.

Perturbations from Atmospheric Drag


Drag is the resistance offered by a gas or liquid to a body moving through it. A spacecraft is subjected to drag forces when
moving through a planet's atmosphere. This drag is greatest during launch and reentry, however, even a space vehicle in low
Earth orbit experiences some drag as it moves through the Earth's thin upper atmosphere. In time, the action of drag on a
space vehicle will cause it to spiral back into the atmosphere, eventually to disintegrate or burn up. If a space vehicle comes
within 120 to 160 km of the Earth's surface, atmospheric drag will bring it down in a few days, with final disintegration occurring
at an altitude of about 80 km. Above approximately 600 km, on the other hand, drag is so weak that orbits usually last more
than 10 years - beyond a satellite's operational lifetime. The deterioration of a spacecraft's orbit due to drag is called decay.
The drag force FD on a body acts in the opposite direction of the velocity vector and is given by the equation

where CD is the drag coefficient, is the air density, v is the body's velocity, and A is the area of the body normal to the flow.
The drag coefficient is dependent on the geometric form of the body and is generally determined by experiment. Earth orbiting
satellites typically have very high drag coefficients in the range of about 2 to 4. Air density is given by the appendix Atmosphere
Properties.
The region above 90 km is the Earth's thermosphere where the absorption of extreme ultraviolet radiation from the Sun results
in a very rapid increase in temperature with altitude. At approximately 200-250 km this temperature approaches a limiting
value, the average value of which ranges between about 700 and 1,400 K over a typical solar cycle. Solar activity also has a
significant affect on atmospheric density, with high solar activity resulting in high density. Below about 150 km the density is not
strongly affected by solar activity; however, at satellite altitudes in the range of 500 to 800 km, the density variations between
solar maximum and solar minimum are approximately two orders of magnitude. The large variations imply that satellites will
decay more rapidly during periods of solar maxima and much more slowly during solar minima.
For circular orbits we can approximate the changes in semi-major axis, period, and velocity per revolution using the following
equations:

where a is the semi-major axis, P is the orbit period, and V, A and m are the satellite's velocity, area, and mass respectively.
The term m/(CDA), called the ballistic coefficient, is given as a constant for most satellites. Drag effects are strongest for
satellites with low ballistic coefficients, this is, light vehicles with large frontal areas.
A rough estimate of a satellite's lifetime, L, due to drag can be computed from

where H is the atmospheric density scale height. A substantially more accurate estimate (although still very approximate) can
be obtained by integrating equation (4.53), taking into account the changes in atmospheric density with both altitude and solar
activity.
Perturbations from Solar Radiation
Solar radiation pressure causes periodic variations in all of the orbital elements. The magnitude of the acceleration in m/s 2
arising from solar radiation pressure is

where A is the cross-sectional area of the satellite exposed to the Sun and m is the mass of the satellite in kilograms. For
satellites below 800 km altitude, acceleration from atmospheric drag is greater than that from solar radiation pressure; above
800 km, acceleration from solar radiation pressure is greater.

Orbit Maneuvers
At some point during the lifetime of most space vehicles or satellites, we must change one or more of the orbital elements. For
example, we may need to transfer from an initial parking orbit to the final mission orbit, rendezvous with or intercept another
spacecraft, or correct the orbital elements to adjust for the perturbations discussed in the previous section. Most frequently, we
must change the orbit altitude, plane, or both. To change the orbit of a space vehicle, we have to change its velocity vector in
magnitude or direction. Most propulsion systems operate for only a short time compared to the orbital period, thus we can treat
the maneuver as an impulsive change in velocity while the position remains fixed. For this reason, any maneuver changing the
orbit of a space vehicle must occur at a point where the old orbit intersects the new orbit. If the orbits do not intersect, we must
use an intermediate orbit that intersects both. In this case, the total maneuver will require at least two propulsive burns.
Orbit Altitude Changes
The most common type of in-plane maneuver changes the size and energy of an orbit, usually from a low-altitude parking orbit
to a higher-altitude mission orbit such as a geosynchronous orbit. Because the initial and final orbits do not intersect, the

maneuver requires a transfer orbit. Figure 4.11 represents a Hohmann transfer orbit. In this case, the transfer orbit's ellipse is
tangent to both the initial and final orbits at the transfer orbit's perigee and apogee respectively. The orbits are tangential, so
the velocity vectors are collinear, and the Hohmann transfer represents the most fuel-efficient transfer between two circular,
coplanar orbits. When transferring from a smaller orbit to a larger orbit, the change in velocity is applied in the direction of
motion; when transferring from a larger orbit to a smaller, the change of velocity is
opposite to the direction of motion.
The total change in velocity required for the orbit transfer is the sum of the velocity
changes at perigee and apogee of the transfer ellipse. Since the velocity vectors are
collinear, the velocity changes are just the differences in magnitudes of the velocities
in each orbit. If we know the initial and final orbits, rA and rB, we can calculate the
total velocity change using the following equations:

Note that equations (4.59) and (4.60) are the same as equation (4.6), and equations (4.61) and (4.62) are the same as
equation (4.45).

Ordinarily we want to transfer a space vehicle using the smallest amount of energy, which usually leads to using a Hohmann
transfer orbit. However, sometimes we may need to transfer a satellite between orbits in less time than that required to
complete the Hohmann transfer. Figure 4.12 shows a faster transfer called the One-Tangent Burn. In this instance the transfer
orbit is tangential to the initial orbit. It intersects the final orbit at an angle equal to the flight path angle of the transfer orbit at
the point of intersection. An infinite number of transfer orbits are tangential to the initial orbit and intersect the final orbit at
some angle. Thus, we may choose the transfer orbit by specifying the size of the transfer orbit, the angular change of the
transfer, or the time required to complete the transfer. We can then define the transfer orbit and calculate the required
velocities.
For example, we may specify the size of the transfer orbit, choosing any semi-major axis that is greater than the semi-major
axis of the Hohmann transfer ellipse. Once we know the semi-major axis of the ellipse, atx, we can calculate the eccentricity,
angular distance traveled in the transfer, the velocity change required for the transfer, and the time required to complete the
transfer. We do this using equations (4.59) through (4.63) and (4.65) above, and the following equations:

Another option for changing the size of an orbit is to use electric propulsion to produce a constant low-thrust burn, which results
in a spiral transfer. We can approximate the velocity change for this type of orbit transfer by

where the velocities are the circular velocities of the two orbits.
Orbit Plane Changes
To change the orientation of a satellite's orbital plane, typically the inclination, we must change the direction of the velocity
vector. This maneuver requires a component of V to be perpendicular to the orbital plane and, therefore, perpendicular to the
initial velocity vector. If the size of the orbit remains constant, the maneuver is called a simple plane change. We can find the
required change in velocity by using the law of cosines. For the case in which Vf is equal to Vi, this expression reduces to

where Vi is the velocity before and after the burn, and


required.

is the angle change

From equation (4.73) we see that if the angular change is equal to 60 degrees, the
required change in velocity is equal to the current velocity. Plane changes are very
expensive in terms of the required change in velocity and resulting propellant
consumption. To minimize this, we should change the plane at a point where the
velocity of the satellite is a minimum: at apogee for an elliptical orbit. In some
cases, it may even be cheaper to boost the satellite into a higher orbit, change the
orbit plane at apogee, and return the satellite to its original orbit.
Typically, orbital transfers require changes in both the size and the plane of the orbit, such as transferring from an inclined
parking orbit at low altitude to a zero-inclination orbit at geosynchronous altitude. We can do this transfer in two steps: a
Hohmann transfer to change the size of the orbit and a simple plane change to make the orbit equatorial. A more efficient
method (less total change in velocity) would be to combine the plane change with the tangential burn at apogee of the transfer
orbit. As we must change both the magnitude and direction of the velocity vector, we can find the required change in velocity
using the law of cosines,

where Vi is the initial velocity, Vf is the final velocity, and


form as equation (4.69).

is the angle change required. Note that equation (4.74) is in the same

As can be seen from equation (4.74), a small plane change can be combined with an altitude change for almost no cost in V or
propellant. Consequently, in practice, geosynchronous transfer is done with a small plane change at perigee and most of the
plane change at apogee.
Another option is to complete the maneuver using three burns. The first burn is a coplanar maneuver placing the satellite into a
transfer orbit with an apogee much higher than the final orbit. When the satellite reaches apogee of the transfer orbit, a

combined plane change maneuver is done. This places the satellite in a second transfer orbit that is coplanar with the final orbit
and has a perigee altitude equal to the altitude of the final orbit. Finally, when the satellite reaches perigee of the second
transfer orbit, another coplanar maneuver places the satellite into the final orbit. This three-burn maneuver may save
propellant, but the propellant savings comes at the expense of the total time required to complete the maneuver.
When a plane change is used to modify inclination only, the magnitude of the angle change is simply the difference between the
initial and final inclinations. In this case, the initial and final orbits share the same ascending and descending nodes. The plane
change maneuver takes places when the space vehicle passes through one of these two nodes.
In some instances, however, a plane change is used to alter an orbit's longitude of ascending node in addition to the inclination.
An example might be a maneuver to correct out-of-plane errors to make the orbits of two space vehicles coplanar in preparation
for a rendezvous. If the orbital elements of the initial and final orbits are known, the plane change angle is determined by the
vector dot product. If ii and i are the inclination and longitude of ascending node of the initial orbit, and if and f are the
inclination and longitude of ascending node of the final orbit, then the angle between the orbital planes, , is given by

The plane change maneuver takes place at one of two nodes where the initial and final orbits intersect. The latitude and
longitude of these nodes are determined by the vector cross product. The position of one of the two nodes is given by

Knowing the position of one node, the second node is simply

Orbit Rendezvous
Orbital transfer becomes more complicated when the object is to rendezvous with or intercept another object in space: both the
interceptor and the target must arrive at the rendezvous point at the same time. This precision demands a phasing orbit to
accomplish the maneuver. A phasing orbit is any orbit that results in the interceptor achieving the desired geometry relative to
the target to initiate a Hohmann transfer. If the initial and final orbits are circular, coplanar, and of different sizes, then the
phasing orbit is simply the initial interceptor orbit. The interceptor remains in the initial orbit until the relative motion between
the interceptor and target results in the desired geometry. At that point, we would inject the interceptor into a Hohmann
transfer orbit.
Launch Windows
Similar to the rendezvous problem is the launch-window problem, or determining the appropriate time to launch from the
surface of the Earth into the desired orbital plane. Because the orbital plane is fixed in inertial space, the launch window is the
time when the launch site on the surface of the Earth rotates through the orbital plane. The time of the launch depends on the
launch site's latitude and longitude and the satellite orbit's inclination and longitude of ascending node.
Orbit Maintenance

Once in their mission orbits, many satellites need no additional orbit adjustment. On the other hand, mission requirements may
demand that we maneuver the satellite to correct the orbital elements when perturbing forces have changed them. Two
particular cases of note are satellites with repeating ground tracks and geostationary satellites.
After the mission of a satellite is complete, several options exist, depending on the orbit. We may allow low-altitude orbits to
decay and reenter the atmosphere or use a velocity change to speed up the process. We may also boost satellites at all altitudes
into benign orbits to reduce the probability of collision with active payloads, especially at synchronous altitudes.
V Budget
To an orbit designer, a space mission is a series of different orbits. For example, a satellite might be released in a low-Earth
parking orbit, transferred to some mission orbit, go through a series of resphasings or alternate mission orbits, and then move
to some final orbit at the end of its useful life. Each of these orbit changes requires energy. The V budget is traditionally used
to account for this energy. It sums all the velocity changes required throughout the space mission life. In a broad sense the V
budget represents the cost for each mission orbit scenario.

The Hyperbolic Orbit


The discussion thus far has focused on the elliptical orbit, which will result whenever a spacecraft has insufficient velocity to
escape the gravity of its primary. There is a velocity, called the escape velocity, Vesc, such that if the spacecraft is launched with
an initial velocity greater than Vesc, it will travel away from the planet and never return. To achieve escape velocity we must give
the spacecraft enough kinetic energy to overcome all of the negative gravitational potential energy. Thus, if m is the mass of the
spacecraft, M is the mass of the planet, and r is the radial distance between the spacecraft and planet, the potential energy is
-GmM /r. The kinetic energy of the spacecraft, when it is launched, is mv2/2. We thus have

which is independent of the mass of the spacecraft.

A space vehicle that has exceeded the escape velocity of a planet will travel a hyperbolic path relative to the planet. The
hyperbola is an unusual and interesting conic section because it has two branches. The arms of a hyperbola are asymptotic to
two intersecting straight line (the asymptotes). If we consider the left-hand focus, f, as the prime focus (where the center of our
gravitating body is located), then only the left branch of the hyperbola represents the possible orbit. If, instead, we assume a
force of repulsion between our satellite and the body located at f (such as the force between two like-charged electric particles),
then the right-hand branch represents the orbit. The parameters a, b and c are labeled in Figure 4.14. We can see that c2 = a2+
b2 for the hyperbola. The eccentricity is,

The angle between the asymptotes, which represents the angle through which the path of a space vehicle is turned by its
encounter with a planet, is labeled . This turning angle is related to the geometry of the hyperbola as follows:

If we let

equal the angle between the periapsis vector and the departure asymptote, i.e. the true anomaly at infinity, we have

If we know the radius, r, velocity, v, and flight path angle, , of a point on the orbit (see Figure 4.15), we can calculate the
eccentricity and semi-major axis using equations (4.30) and (4.32) as previously presented. Note that the semi-major axis of a
hyperbola is negative.
The true anomaly corresponding to known valves of r, v and can be calculated using equation (4.31), however special care
must be taken to assure the angle is placed in the correct quadrant. It may be easier to first calculate e and a, and then
calculate true anomaly using equation (4.43), rearranged as follows:

Whenever

is positive,

should be taken as positive; whenever

is negative,

should be taken as negative.

The impact parameter, b, is the distance of closest approach that would result between a spacecraft and planet if the spacecraft
trajectory was undeflected by gravity. The impact parameter is,

Closet approach occurs at periapsis, where the radius distance, ro, is equal to

p is a geometrical constant of the conic called the parameter or semi-latus rectum, and is equal to

At any known true anomaly, the magnitude of a spacecraft's radius vector, its flight-path angle, and its velocity can be
calculated using equations (4.43), (4.44) and (4.45).
Early we introduced the variable eccentric anomaly and its use in deriving the time of flight in an elliptical orbit. In a similar
manner, the analytical derivation of the hyperbolic time of flight, using the hyperbolic eccentric anomaly, F, can be derived as
follows:

where,

Whenever

is positive, F should be taken as positive; whenever

is negative, F should be taken as negative.

Hyperbolic Excess Velocity


If you give a space vehicle exactly escape velocity, it will just barely escape the gravitational field, which means that its velocity
will be approaching zero as its distance from the force center approaches infinity. If, on the other hand, we give our vehicle
more than escape velocity at a point near Earth, we would expect the velocity at a great distance from Earth to be approaching
some finite constant value. This residual velocity the vehicle would have left over even at infinity is called hyperbolic excess
velocity. We can calculate this velocity from the energy equation written for two points on the hyperbolic escape trajectory a
point near Earth called the burnout point and a point an infinite distance from Earth where the velocity will be the hyperbolic
excess velocity, v. Solving for v we obtain

Note that if v = 0 (as it is on a parabolic trajectory), the burnout velocity,


vbo, becomes simply the escape velocity.

It is, of course, absurd to talk about a space vehicle "reaching infinity" and in this sense it is meaningless to talk about escaping
a gravitational field completely. It is a fact, however, that once a space vehicle is a great distance from Earth, for all practical
purposes it has escaped. In other words, it has already slowed down to very nearly its hyperbolic excess velocity. It is
convenient to define a sphere around every gravitational body and say that when a probe crosses the edge of this sphere of
influence it has escaped. Although it is difficult to get agreement on exactly where the sphere of influence should be drawn, the
concept is convenient and is widely used, especially in lunar and interplanetary trajectories. For most purposes, the radius of the
sphere of influence for a planet can be calculated as follows:

where Dsp is the distance between the Sun and the planet, Mp is the mass of the planet, and Ms is the mass of the Sun. Equation
(4.89) is also valid for calculating a moon's sphere of influence, where the moon is substituted for the planet and the planet for
the Sun.

ATMOSPHERE PROPERTIES
Physical Properties of U.S. Standard Atmosphere, 1976 in SI Units
Altitude
(meters)

Temperature
(K)

Pressure
(Pa)

Density
(kg/m3)

Viscosity
(N-s/m2)

-2,000

301.15

1.27774E+5

1.47808

1.87630E-5

-1,000

294.65

1.13929E+5

1.34700

1.84434E-5

288.15

1.01325E+5

1.22500

1.81206E-5

1,000

281.65

8.98746E+4

1.11164

1.77943E-5

2,000

275.15

7.94952E+4

1.00649

1.74645E-5

3,000

268.65

7.01085E+4

9.09122E-1

1.71311E-5

4,000

262.15

6.16402E+4

8.19129E-1

1.67940E-5

5,000

255.65

5.40199E+4

7.36116E-1

1.64531E-5

6,000

249.15

4.71810E+4

6.59697E-1

1.61084E-5

7,000

242.65

4.10607E+4

5.89501E-1

1.57596E-5

8,000

236.15

3.55998E+4

5.25168E-1

1.54068E-5

9,000

229.65

3.07425E+4

4.66348E-1

1.50498E-5

10,000

223.15

2.64363E+4

4.12707E-1

1.46884E-5

12,000

216.65

1.93304E+4

3.10828E-1

1.43226E-5

15,000

216.65

1.20446E+4

1.93674E-1

1.43226E-5

20,000

216.65

5.47489E+3

8.80349E-2

1.43226E-5

25,000

221.65

2.51102E+3

3.94658E-2

1.46044E-5

30,000

226.65

1.17187E+3

1.80119E-2

1.48835E-5

35,000

237.05

5.58924E+2

8.21392E-3

1.54559E-5

40,000

251.05

2.77522E+2

3.85101E-3

1.62096E-5

45,000

265.05

1.43135E+2

1.88129E-3

1.69449E-5

50,000

270.65

7.59448E+1

9.77525E-4

1.72341E-5

60,000

245.45

2.03143E+1

2.88321E-4

1.59104E-5

70,000

217.45

4.63422

7.42430E-5

1.43679E-5

80,000

196.65

8.86280E-1

1.57005E-5

1.31682E-5

84,852

186.95

3.73384E-1

6.95788E-6

1.25915E-5

Basic Assumptions
Air is a clean, dry, perfect gas mixture; Specific heat ratio = 1.40; Molecular weight to 86 km = 28.9644
Principal sea-level constituents: N278.084%, O220.9476%, Ar0.934%, CO20.0314%, Ne0.001818%, He0.000524%, CH4
0.0002%
See bottom of page for U.S. units

MSISE-90 Model of Earth's Upper Atmosphere


Altitude
(km)

Low Solar Activity


Temp.
(K)

Density
(kg/m3)

300.2511

1.17E+00

1.01E+05

20

206.2085

9.48E-02

40

257.6979

60
80

Mean Solar Activity

Pressure Mol. Wt.


(Pa)
(kg/kmol)

Temp.
(K)

Density
(kg/m3)

28.9502

300.2511

1.17E+00

1.01E+05

5.62E+03

28.9502

206.2085

9.49E-02

4.07E-03

3.01E+02

28.9502

257.6979

244.1212

3.31E-04

2.32E+01

28.9502

203.1065

1.69E-05

9.81E-01

29.1353

Extremely High Solar Activity

Pressure Mol. Wt.


(Pa)
(kg/kmol)

Temp.
(K)

Density
(kg/m3)

Pressure Mol. Wt.


(Pa)
(kg/kmol)

28.9502

300.2511

1.16E+00

9.98E+04

28.9502

5.62E+03

28.9502

206.2085

9.41E-02

5.57E+03

28.9502

4.07E-03

3.02E+02

28.9502

257.6979

4.04E-03

2.99E+02

28.9502

244.1212

3.31E-04

2.32E+01

28.9502

244.1212

3.28E-04

2.30E+01

28.9502

196.3636

1.68E-05

9.45E-01

29.0175

172.2146

1.68E-05

8.42E-01

28.5290

100

168.7219

5.77E-07

2.89E-02

28.0036

184.0160

5.08E-07

2.81E-02

27.7137

297.3338

2.78E-07

2.63E-02

26.1997

120

356.8669

1.70E-08

1.92E-03

26.3948

374.9715

1.80E-08

2.17E-03

25.8745

430.8385

2.34E-08

3.55E-03

23.6456

140

545.8594

2.96E-09

5.37E-04

25.0665

635.5703

3.26E-09

7.03E-04

24.5349

875.9174

4.93E-09

1.61E-03

22.3209

160

630.0652

9.65E-10

2.13E-04

23.7884

787.5532

1.18E-09

3.31E-04

23.4225

1,143.5426

2.23E-09

9.90E-04

21.4577

180

667.8662

3.90E-10

9.62E-05

22.5037

877.6729

5.51E-10

1.80E-04

22.4106

1,314.3427

1.28E-09

6.76E-04

20.7706

200

684.9187

1.75E-10

4.70E-05

21.2516

931.2806

2.91E-10

1.05E-04

21.4734

1,423.6469

8.28E-10

4.86E-04

20.1836

220

692.6487

8.47E-11

2.43E-05

20.0935

963.2701

1.66E-10

6.44E-05

20.6108

1,493.7864

5.69E-10

3.60E-04

19.6664

240

696.1697

4.31E-11

1.31E-05

19.0789

982.4191

9.91E-11

4.09E-05

19.8292

1,538.9154

4.08E-10

2.72E-04

19.2046

260

697.7811

2.30E-11

7.31E-06

18.2300

993.9173

6.16E-11

2.66E-05

19.1337

1,568.0294

3.00E-10

2.08E-04

18.7901

280

698.5220

1.27E-11

4.20E-06

17.5402 1,000.8427

3.94E-11

1.77E-05

18.5256

1,586.8613

2.25E-10

1.61E-04

18.4178

300

698.8644

7.22E-12

2.47E-06

16.9830 1,005.0267

2.58E-11

1.20E-05

18.0015

1,599.0743

1.71E-10

1.26E-04

18.0839

320

699.0233

4.21E-12

1.48E-06

16.5214 1,007.5620

1.72E-11

8.20E-06

17.5537

1,607.0154

1.32E-10

9.93E-05

17.7852

340

699.0973

2.50E-12

9.01E-07

16.1147

1,009.1030

1.16E-11

5.69E-06

17.1721

1,612.1920

1.03E-10

7.86E-05

17.5186

360

699.1320

1.51E-12

5.57E-07

15.7219 1,010.0423

7.99E-12

3.98E-06

16.8449

1,615.5751

8.05E-11

6.26E-05

17.2812

380

699.1483

9.20E-13

3.50E-07

15.3028 1,010.6166

5.55E-12

2.81E-06

16.5597

1,617.7916

6.35E-11

5.01E-05

17.0699

400

699.1561

5.68E-13

2.23E-07

14.8185 1,010.9688

3.89E-12

2.01E-06

16.3044

1,619.2476

5.04E-11

4.02E-05

16.8818

420

699.1597

3.54E-13

1.45E-07

14.2332

1,011.1853

2.75E-12

1.44E-06

16.0669

1,620.2062

4.02E-11

3.25E-05

16.7142

440

699.1615

2.23E-13

9.61E-08

13.5181

1,011.3190

1.96E-12

1.04E-06

15.8360

1,620.8390

3.23E-11

2.63E-05

16.5643

460

699.1623

1.42E-13

6.54E-08

12.6581

1,011.4014

1.40E-12

7.55E-07

15.6008

1,621.2577

2.60E-11

2.13E-05

16.4297

480

699.1627

9.20E-14

4.59E-08

11.6594

1,011.4526

1.01E-12

5.53E-07

15.3508

1,621.5354

2.10E-11

1.73E-05

16.3079

500

699.1629

6.03E-14

3.32E-08

10.5547

1,011.4845

7.30E-13

4.07E-07

15.0760

1,621.7200

1.70E-11

1.42E-05

16.1967

520

699.1630

4.03E-14

2.49E-08

9.4006

1,011.5043

5.31E-13

3.03E-07

14.7669

1,621.8430

1.38E-11

1.16E-05

16.0940

540

699.1630

2.75E-14

1.94E-08

8.2657

1,011.5168

3.88E-13

2.27E-07

14.4148

1,621.9253

1.13E-11

9.50E-06

15.9980

560

699.1631

1.93E-14

1.55E-08

7.2141

1,011.5245

2.85E-13

1.71E-07

14.0125

1,621.9803

9.21E-12

7.81E-06

15.9067

580

699.1631

1.39E-14

1.28E-08

6.2904

1,011.5294

2.11E-13

1.31E-07

13.5547

1,622.0172

7.55E-12

6.44E-06

15.8187

600

699.1631

1.03E-14

1.09E-08

5.5149

1,011.5325

1.56E-13

1.01E-07

13.0389

1,622.0421

6.20E-12

5.31E-06

15.7321

620

699.1631

7.90E-15

9.40E-09

4.8864

1,011.5345

1.17E-13

7.89E-08

12.4665

1,622.0588

5.10E-12

4.40E-06

15.6457

640

699.1631

6.24E-15

8.27E-09

4.3891

1,011.5357

8.79E-14

6.24E-08

11.8428

1,622.0702

4.20E-12

3.65E-06

15.5578

660

699.1631

5.06E-15

7.36E-09

4.0012

1,011.5365

6.65E-14

5.01E-08

11.1779

1,622.0778

3.47E-12

3.03E-06

15.4672

680

699.1631

4.21E-15

6.62E-09

3.6999

1,011.5370

5.08E-14

4.07E-08

10.4854

1,622.0830

2.88E-12

2.52E-06

15.3725

700

699.1631

3.58E-15

6.00E-09

3.4648

1,011.5374

3.91E-14

3.36E-08

9.7818

1,622.0865

2.38E-12

2.11E-06

15.2723

720

699.1631

3.09E-15

5.48E-09

3.2789

1,011.5375

3.04E-14

2.82E-08

9.0847

1,622.0890

1.98E-12

1.76E-06

15.1653

740

699.1631

2.70E-15

5.02E-09

3.1289

1,011.5377

2.39E-14

2.39E-08

8.4111

1,622.0906

1.65E-12

1.48E-06

15.0503

760

699.1631

2.39E-15

4.63E-09

3.0049

1,011.5377

1.90E-14

2.06E-08

7.7753

1,622.0918

1.37E-12

1.24E-06

14.9260

780

699.1631

2.13E-15

4.28E-09

2.8996

1,011.5378

1.53E-14

1.79E-08

7.1884

1,622.0925

1.15E-12

1.05E-06

14.7912

800

699.1631

1.91E-15

3.96E-09

2.8075

1,011.5378

1.25E-14

1.58E-08

6.6572

1,622.0930

9.59E-13

8.84E-07

14.6447

820

699.1631

1.73E-15

3.68E-09

2.7249

1,011.5378

1.03E-14

1.40E-08

6.1849

1,622.0934

8.04E-13

7.48E-07

14.4854

840

699.1631

1.56E-15

3.43E-09

2.6492

1,011.5379

8.64E-15

1.26E-08

5.7711

1,622.0936

6.74E-13

6.36E-07

14.3123

860

699.1631

1.42E-15

3.21E-09

2.5784

1,011.5379

7.32E-15

1.14E-08

5.4132

1,622.0939

5.67E-13

5.42E-07

14.1244

880

699.1631

1.30E-15

3.00E-09

2.5113

1,011.5379

6.28E-15

1.04E-08

5.1066

1,622.0940

4.77E-13

4.63E-07

13.9210

900

699.1631

1.18E-15

2.81E-09

2.4470

1,011.5379

5.46E-15

9.47E-09

4.8460

1,622.0940

4.03E-13

3.97E-07

13.7015

Atmospheric Scale Height & Density, to 35,786 km


Altitude
(km)

Scale Height
(km)

Atmospheric Density
Mean
(kg/m3)

Maximum
(kg/m3)

8.4

1.225

1.225

100

5.9

5.25E-7

5.75E-7

150

25.5

1.73E-9

1.99E-9

200

37.5

2.41E-10

3.65E-10

250

44.8

5.97E-11

1.20E-10

300

50.3

1.87E-11

4.84E-11

350

54.8

6.66E-12

2.18E-11

400

58.2

2.62E-12

1.05E-11

450

61.3

1.09E-12

5.35E-12

500

64.5

4.76E-13

2.82E-12

550

68.7

2.14E-13

1.53E-12

600

74.8

9.89E-14

8.46E-13

650

84.4

4.73E-14

4.77E-13

700

99.3

2.36E-14

2.73E-13

750

121

1.24E-14

1.59E-13

800

151

6.95E-15

9.41E-14

850

188

4.22E-15

5.67E-14

900

226

2.78E-15

3.49E-14

950

263

1.98E-15

2.21E-14

1,000

296

1.49E-15

1.43E-14

1,250

408

5.70E-16

2.82E-15

1,500

516

2.79E-16

1.16E-15

2,000

829

9.09E-17

3.80E-16

2,500

1220

4.23E-17

1.54E-16

3,000

1590

2.54E-17

7.09E-17

3,500

1900

1.77E-17

3.67E-17

4,000

2180

1.34E-17

2.11E-17

4,500

2430

1.06E-17

1.34E-17

5,000

2690

8.62E-18

9.30E-18

6,000

3200

6.09E-18

5.41E-18

7,000

3750

4.56E-18

3.74E-18

8,000

4340

3.56E-18

2.87E-18

9,000

4970

2.87E-18

2.34E-18

10,000

5630

2.37E-18

1.98E-18

15,000

9600

1.21E-18

1.16E-18

20,000

14600

7.92E-19

8.42E-19

25,000

20700

5.95E-19

6.81E-19

30,000

27800

4.83E-19

5.84E-19

35,000

36000

4.13E-19

5.21E-19

35,786

37300

4.04E-19

5.12E-19

Physical Properties of U.S. Standard Atmosphere, 1976 in U.S. Units


Altitude
(feet)

Temperature
(oR)

Pressure
(psia)

Density
(slug/ft3)

Viscosity
(lb-s/ft2)

-6,000

540.07

18.1749

2.82344E-3

3.90735E-7

-3,000

529.37

16.3607

2.59297E-3

3.84625E-7

518.67

14.6959

2.37717E-3

3.78456E-7

5,000

500.84

12.2277

2.04834E-3

3.68041E-7

10,000

483.01

10.1065

1.75549E-3

3.57454E-7

15,000

465.18

8.29354

1.49581E-3

3.46688E-7

20,000

447.35

6.75343

1.26659E-3

3.35735E-7

25,000

429.52

5.45355

1.06526E-3

3.24588E-7

30,000

411.69

4.36413

8.89378E-4

3.13240E-7

35,000

393.85

3.45803

7.36627E-4

3.01681E-7

40,000

389.97

2.72003

5.85189E-4

2.99135E-7

45,000

389.97

2.13897

4.60180E-4

2.99135E-7

50,000

389.97

1.68204

3.61876E-4

2.99135E-7

55,000

389.97

1.32272

2.84571E-4

2.99135E-7

60,000

389.97

1.04016

2.23781E-4

2.99135E-7

70,000

392.37

6.43641E-1

1.37625E-4

3.00712E-7

80,000

397.86

4.00519E-1

8.44590E-5

3.04297E-7

100,000

408.83

1.58114E-1

3.24473E-5

3.11406E-7

120,000

434.63

6.47290E-2

1.24948E-5

3.27808E-7

140,000

465.36

2.81301E-2

5.07154E-6

3.46797E-7

160,000

487.17

1.28682E-2

2.21610E-6

3.59941E-7

180,000

467.70

5.90183E-3

1.05871E-6

3.48219E-7

200,000

436.97

2.57592E-3

4.94576E-7

3.29273E-7

225,000

398.57

8.38438E-4

1.76492E-7

3.04757E-7

250,000

367.65

2.45671E-4

5.60626E-8

2.84296E-7

278,386

336.50

5.41542E-5

1.35020E-8

2.62978E-7

EXAMPLE PROBLEMS
PROBLEM 1.1
A spacecraft's engine ejects mass at a rate of 30 kg/s with an exhaust velocity
of 3,100 m/s. The pressure at the nozzle exit is 5 kPa and the exit area is
0.7 m2. What is the thrust of the engine in a vacuum?

SOLUTION,

Given:

q = 30 kg/s
Ve = 3,100 m/s
Ae = 0.7 m2
Pe = 5 kPa = 5,000 N/m2
Pa = 0

Equation (1.6),
F = q Ve + (Pe - Pa) Ae
F = 30 3,100 + (5,000 - 0) 0.7
F = 96,500 N

PROBLEM 1.2
The spacecraft in problem 1.1 has an initial mass of 30,000 kg. What is the
change in velocity if the spacecraft burns its engine for one minute?

SOLUTION,
Given:

M = 30,000 kg
q = 30 kg/s
Ve = 3,100 m/s
t = 60 s

Equation (1.16),
V = Ve LN[ M / (M - qt) ]
V = 3,100 LN[ 30,000 / (30,000 - (30 60)) ]
V = 192 m/s

PROBLEM 1.3
A spacecraft's dry mass is 75,000 kg and the effective exhaust gas velocity
of its main engine is 3,100 m/s. How much propellant must be carried if the
propulsion system is to produce a total
v of 700 m/s?

SOLUTION,
Given:

Mf = 75,000 kg
C = 3,100 m/s
V = 700 m/s

Equation (1.20),
Mo = Mf e(V / C)
Mo = 75,000 e(700
Mo = 94,000 kg

/ 3,100)

Propellant mass,
Mp = Mo - Mf
Mp = 94,000 - 75,000
Mp = 19,000 kg

PROBLEM 1.4
A 5,000 kg spacecraft is in Earth orbit traveling at a velocity of 7,790 m/s.
Its engine is burned to accelerate it to a velocity of 12,000 m/s placing it

on an escape trajectory. The engine expels mass at a rate of 10 kg/s and an


effective velocity of 3,000 m/s. Calculate the duration of the burn.

SOLUTION,
Given:

M = 5,000 kg
q = 10 kg/s
C = 3,000 m/s
V = 12,000 - 7,790 = 4,210 m/s

Equation (1.21),
t = M / q [ 1 - 1 / e(V / C) ]
t = 5,000 / 10 [ 1 - 1 / e(4,210
t = 377 s

/ 3,000)

PROBLEM 1.5
A rocket engine burning liquid oxygen and kerosene operates at a mixture ratio
of 2.26 and a combustion chamber pressure of 50 atmospheres. If the nozzle is
expanded to operate at sea level, calculate the exhaust gas velocity relative
to the rocket.

SOLUTION,
Given:

O/F = 2.26
Pc = 50 atm
Pe = Pa = 1 atm

From LOX/Kerosene Charts we estimate,

Tc = 3,470 K
M = 21.40
k = 1.221
Equation (1.22),
Ve = SQRT[ (2 k / (k - 1)) (R* Tc / M) (1 - (Pe / Pc)(k-1)/k) ]
Ve = SQRT[ (2 1.221 / (1.221 - 1)) (8,314.46 3,470 / 21.40) (1 - (1 / 50)(1.221-1)/1.221) ]
Ve = 2,749 m/s

PROBLEM 1.6
A rocket engine produces a thrust of 1,000 kN at sea level with a propellant
flow rate of 400 kg/s. Calculate the specific impulse.

SOLUTION,
Given:

F = 1,000,000 N
q = 400 kg/s

Equation (1.23),
Isp = F / (q g)
Isp = 1,000,000 / (400 9.80665)
Isp = 255 s (sea level)

PROBLEM 1.7
A rocket engine uses the same propellant, mixture ratio, and combustion chamber
pressure as that in problem 1.5. If the propellant flow rate is 500 kg/s,
calculate the area of the exhaust nozzle throat.

SOLUTION,
Given:

Pc = 50 0.101325 = 5.066 MPa


Tc = 3,470<sup.o< sup=""> K
M = 21.40
k = 1.221
q = 500 kg/s

Equation (1.27),
Pt = Pc [1 + (k - 1) / 2]-k/(k-1)
Pt = 5.066 [1 + (1.221 - 1) / 2]-1.221/(1.221-1)
Pt = 2.839 MPa = 2.839106 N/m2
Equation (1.28),
Tt = Tc / (1 + (k - 1) / 2)
Tt = 3,470 / (1 + (1.221 - 1) / 2)
Tt = 3,125 K
Equation (1.26),
At = (q / Pt) SQRT[ (R* Tt) / (M k) ]
At = (500 / 2.839106) SQRT[ (8,314.46 3,125) / (21.40 1.221) ]
At = 0.1756 m2

</sup.o<>

PROBLEM 1.8
The rocket engine in problem 1.7 is optimized to operate at an elevation of 2000
meters. Calculate the area of the nozzle exit and the section ratio.

SOLUTION,
Given:

Pc = 5.066 MPa
At = 0.1756 m2
k = 1.221

From Atmosphere Properties,


Pa = 0.0795 MPa
Equation (1.29),
Nm2 = (2 / (k - 1)) [(Pc / Pa)(k-1)/k - 1]
Nm2 = (2 / (1.221 - 1)) [(5.066 / 0.0795)(1.221-1)/1.221 - 1]
Nm2 = 10.15
Nm = (10.15)1/2 = 3.185
Equation (1.30),
Ae = (At / Nm) [(1 + (k - 1) / 2 Nm2)/((k + 1) / 2)](k+1)/(2(k-1))
Ae = (0.1756 / 3.185) [(1 + (1.221 - 1) / 2 10.15)/((1.221 + 1) / 2)](1.221+1)/(2(1.221-1))
Ae = 1.426 m2
Section Ratio,
Ae / At = 1.426 / 0.1756 = 8.12

PROBLEM 1.9
For the rocket engine in problem 1.7, calculate the volume and dimensions of a
possible combustion chamber. The convergent cone half-angle is 20 degrees.

SOLUTION,
Given:

At = 0.1756 m2 = 1,756 cm2


Dt = 2 (1,756/ )1/2 = 47.3 cm
= 20o

From Table 1,
L* = 102-127 cm for LOX/RP-1, let's use 110 cm
Equation (1.33),
Vc = At L*
Vc = 1,756 110 = 193,160 cm3
From Figure 1.7,
Lc = 66 cm (second-order approximation)
Equation (1.35),
Dc = SQRT[(Dt3 + 24/
tan
Vc) / (Dc + 6 tan
Lc)]
Dc = SQRT[(47.33 + 24/
tan(20) 193,160) / (Dc + 6 tan(20) 66)]
Dc = 56.6 cm (four interations)

PROBLEM 1.10
A solid rocket motor burns along the face
meters long and 1 meter in diameter. The
of 5.5, a pressure exponent of 0.4, and a
burn rate and the product generation rate

of a central cylindrical channel 10


propellant has a burn rate coefficient
density of 1.70 g/ml. Calculate the
when the chamber pressure is 5.0 MPa.

SOLUTION,
Given:

a = 5.5
n = 0.4
Pc = 5.0 MPa
p = 1.70 g/ml
Ab =
1 10 = 31.416 m2

Equation (1.36),
r = a Pcn
r = 5.5 5.00.4 = 10.47 mm/s
Equation (1.37),
q = p Ab r
q = 1.70 31.416 10.47 = 559 kg/s

PROBLEM 1.11
Calculate the ideal density of a solid rocket propellant consisting of 68%
ammonium perchlorate, 18% aluminum, and 14% HTPB by mass.

SOLUTION,
Given:

wAP = 0.68
wAl = 0.18
wHTPB = 0.14

From Properties of Rocket Propellants we have,


= 1.95 g/ml
Al = 2.70 g/ml
HTPB = 0.93 g/ml
AP

Equation (1.38),
p = 1 /
)i
i (w /
p = 1 / [(0.68 / 1.95) + (0.18 / 2.70) + (0.14 / 0.93)]
p = 1.767

PROBLEM 1.12
A two-stage rocket has
kg, 1st-stage dry mass
dry mass 3,000 kg, and
1st and 2nd stages are
total
V.

SOLUTION,

the following masses: 1st-stage propellant mass 120,000


9,000 kg, 2nd-stage propellant mass 30,000 kg, 2nd-stage
payload mass 3,000 kg. The specific impulses of the
260 s and 320 s respectively. Calculate the rocket's

Given:

Mo1 = 120,000 + 9,000 + 30,000 + 3,000 + 3,000 = 165,000 kg


Mf1 = 9,000 + 30,000 + 3,000 + 3,000 = 45,000 kg
Isp1 = 260 s
Mo2 = 30,000 + 3,000 + 3,000 = 36,000 kg
Mf2 = 3,000 + 3,000 = 6,000 kg
Isp2 = 320 s

Equation (1.24),
C1 = Isp1g
C1 = 260 9.80665 = 2,550 m/s
C2 = Isp2g
C2 = 320 9.80665 = 3,138 m/s
Equation (1.39),
V1 = C1 LN[ Mo1 / Mf1 ]
V1 = 2,550 LN[ 165,000 / 45,000 ]
V1 = 3,313 m/s
V2 = C2 LN[ Mo2 / Mf2 ]
V2 = 3,138 LN[ 36,000 / 6,000 ]
V2 = 5,623 m/s
Equation (1.40),
VTotal =
V1 +
V2
VTotal = 3,313 + 5,623
VTotal = 8,936 m/s

PROBLEM 4.1
Calculate the velocity of an artificial satellite orbiting the Earth in a
circular orbit at an altitude of 200 km above the Earth's surface.

SOLUTION,
From Basics Constants,
Radius of Earth = 6,378.14 km
GM of Earth = 3.9860051014 m3/s2
Given:

r = (6,378.14 + 200) 1,000 = 6,578,140 m

Equation (4.6),
v = SQRT[ GM / r ]
v = SQRT[ 3.9860051014 / 6,578,140 ]
v = 7,784 m/s

PROBLEM 4.2
Calculate the period of revolution for the satellite in problem 4.1.

SOLUTION,
Given:

r = 6,578,140 m

Equation (4.9),

P2 = 4

r3 / GM

P = SQRT[ 4
P = SQRT[ 4
P = 5,310 s

2
2

r3 / GM ]
6,578,1403 / 3.9860051014 ]

PROBLEM 4.3
Calculate the radius of orbit for a Earth satellite in a geosynchronous orbit,
where the Earth's rotational period is 86,164.1 seconds.

SOLUTION,
Given:

P = 86,164.1 s

Equation (4.9),
P2 = 4

r3 / GM

2
r = [ P2 GM / (4
) ]1/3
2
r = [ 86,164.1 3.9860051014 / (4
r = 42,164,170 m

PROBLEM 4.4

) ]1/3

An artificial Earth satellite is in an elliptical orbit which brings it to


an altitude of 250 km at perigee and out to an altitude of 500 km at apogee.
Calculate the velocity of the satellite at both perigee and apogee.

SOLUTION,
Given:

Rp = (6,378.14 + 250) 1,000 = 6,628,140 m


Ra = (6,378.14 + 500) 1,000 = 6,878,140 m

Equations (4.16) and (4.17),


Vp = SQRT[ 2 GM Ra / (Rp (Ra + Rp)) ]
Vp = SQRT[ 2 3.9860051014 6,878,140 / (6,628,140 (6,878,140 + 6,628,140)) ]
Vp = 7,826 m/s
Va = SQRT[ 2 GM Rp / (Ra (Ra + Rp)) ]
Va = SQRT[ 2 3.9860051014 6,628,140 / (6,878,140 (6,878,140 + 6,628,140)) ]
Va = 7,542 m/s

PROBLEM 4.5
A satellite in Earth orbit passes through its perigee point at an altitude of
200 km above the Earth's surface and at a velocity of 7,850 m/s. Calculate the
apogee altitude of the satellite.

SOLUTION,
Given:

Rp = (6,378.14 + 200) 1,000 = 6,578,140 m


Vp = 7,850 m/s

Equation (4.18),
Ra = Rp / [2 GM / (Rp Vp2) - 1]
Ra = 6,578,140 / [2 3.9860051014 / (6,578,140 7,8502) - 1]
Ra = 6,805,140 m
Altitude @ apogee = 6,805,140 / 1,000 - 6,378.14 = 427.0 km

PROBLEM 4.6
Calculate the eccentricity of the orbit for the satellite in problem 4.5.

SOLUTION,
Given:

Rp = 6,578,140 m
Vp = 7,850 m/s

Equation (4.20),
e = Rp Vp2 / GM - 1
e = 6,578,140 7,8502 / 3.9860051014 - 1
e = 0.01696

PROBLEM 4.7
A satellite in Earth orbit has a semi-major axis of 6,700 km and an eccentricity
of 0.01. Calculate the satellite's altitude at both perigee and apogee.

SOLUTION,
Given:

a = 6,700 km
e = 0.01

Equation (4.21) and (4.22),


Rp = a (1 - e)
Rp = 6,700 (1 - .01)
Rp = 6,633 km
Altitude @ perigee = 6,633 - 6,378.14 = 254.9 km
Ra = a (1 + e)
Ra = 6,700 (1 + .01)
Ra = 6,767 km
Altitude @ apogee = 6,767 - 6,378.14 = 388.9 km

PROBLEM 4.8
A satellite is launched into Earth orbit where its launch vehicle burns out at
an altitude of 250 km. At burnout the satellite's velocity is 7,900 m/s with the
zenith angle equal to 89 degrees. Calculate the satellite's altitude at perigee
and apogee.

SOLUTION,
Given:

r1 = (6,378.14 + 250) 1,000 = 6,628,140 m

v1 = 7,900 m/s
= 89o
Equation (4.26),
(Rp / r1)1,2 = ( -C SQRT[ C2 - 4 (1 - C) -sin2
where

]) / (2 (1 - C))

C = 2 GM / (r1 v12)
C = 2 3.9860051014 / (6,628,140 7,9002)
C = 1.927179

(Rp / r1)1,2 = ( -1.927179 SQRT[ 1.9271792 - 4 -0.927179 -sin2(89) ]) / (2 -0.927179)


(Rp / r1)1,2 = 0.996019 and 1.082521
Perigee Radius, Rp = Rp1 = r1 (Rp / r1)1
Rp = 6,628,140 0.996019
Rp = 6,601,750 m
Altitude @ perigee = 6,601,750 / 1,000 - 6,378.14 = 223.6 km
Apogee Radius, Ra = Rp2 = r1 (Rp / r1)2
Ra = 6,628,140 1.082521
Ra = 7,175,100 m
Altitude @ agogee = 7,175,100 / 1,000 - 6,378.14 = 797.0 km

PROBLEM 4.9
Calculate the eccentricity of the orbit for the satellite in problem 4.8.

SOLUTION,
Given:

r1 = 6,628,140 m
v1 = 7,900 m/s
= 89o

Equation (4.27),
e = SQRT[ (r1 v12 / GM - 1)2 sin2
+ cos2
]
2
e = SQRT[ (6,628,140 7,900 / 3.9860051014 - 1)2 sin2(89) + cos2(89) ]
e = 0.0416170

PROBLEM 4.10
Calculate the angle
in problem 4.8.

from perigee point to launch point for the satellite

SOLUTION,
Given:

r1 = 6,628,140 m
v1 = 7,900 m/s
= 89o

Equation (4.28),
tan
tan
tan

= (r1 v12 / GM) sin


cos
/ [(r1 v12 / GM) sin2
2
14
= (6,628,140 7,900 / 3.98600510 ) sin(89) cos(89)
/ [(6,628,140 7,9002 / 3.9860051014) sin2(89) - 1]
= 0.48329

- 1]

= arctan(0.48329)
= 25.794o

PROBLEM 4.11
Calculate the semi-major axis of the orbit for the satellite in problem 4.8.

SOLUTION,
Given:

r1 = 6,628,140 m
v1 = 7,900 m/s

Equation (4.32),
a = 1 / ( 2 / r1 - v12 / GM )
a = 1 / ( 2 / 6,628,140 - 7,9002 / 3.9860051014) )
a = 6,888,430 m

PROBLEM 4.12
For the satellite in problem 4.8, burnout occurs 2000-10-20, 15:00 UT. The
geocentric coordinates at burnout are 32o N latitude, 60o W longitude, and the
azimuth heading is 86o. Calculate the orbit's inclination, argument of perigee,
and longitude of ascending node.

SOLUTION,
Given:

= 86o
= 32o
o
2 = -60

From problem 4.10,


= 25.794o
Equation (4.33),
cos(i) = cos( ) sin( )
cos(i) = cos(32) sin(86)
i = 32.223o
Equations (4.34) and (4.36),
tan( ) = tan( ) / cos( )
tan( ) = tan(32) / cos(86)
= 83.630o
=
= 83.630 - 25.794
= 57.836o
Equations (4.35) and (4.37),
tan(
) = sin( ) tan( )
tan(
) = sin(32) tan(86)
= 82.483o
1
1

= 2 = -60 - 82.483

= -142.483o
= Sidereal time at -142.483 longitude, 2000-10-20, 15:00 UT
= 7h 27' 34" = 111.892o

PROBLEM 4.13
A satellite is in an orbit with a semi-major axis of 7,500 km and an eccentricity
of 0.1. Calculate the time it takes to move from a position 30 degrees past
perigee to 90 degrees past perigee.

SOLUTION,
Given:

a = 7,500 1,000 = 7,500,000 m


e = 0.1
tO = 0
/180 = 0.52360 radians
O = 30 deg
= 90 deg
/180 = 1.57080 radians

Equation (4.40),
cos E = (e + cos

) / (1 + e cos

Eo = arccos[(0.1 + cos(0.52360)) / (1 + 0.1 cos(0.52360))]


Eo = 0.47557 radians
E = arccos[(0.1 + cos(1.57080)) / (1 + 0.1 cos(1.57080))]
E = 1.47063 radians
Equation (4.41),

M = E - e sin E
Mo = 0.47557 - 0.1 sin(0.47557)
Mo = 0.42978 radians
M = 1.47063 - 0.1 sin(1.47063)
M = 1.37113 radians
Equation (4.39),
n = SQRT[ GM / a3 ]
n = SQRT[ 3.9860051014 / 7,500,0003 ]
n = 0.00097202 rad/s
Equation (4.38),
M - Mo = n (t - tO)
t = tO + (M - Mo) / n
t = 0 + (1.37113 - 0.42978) / 0.00097202
t = 968.4 s

PROBLEM 4.14
The satellite in problem 4.13 has a true anomaly of 90 degrees.
satellite's position, i.e. it's true anomaly, 20 minutes later?

SOLUTION,
Given:

a = 7,500,000 m
e = 0.1
tO = 0
t = 20 60 = 1,200 s

What will be the

= 90

/180 = 1.57080 rad

From problem 4.13,


Mo = 1.37113 rad
n = 0.00097202 rad/s
Equation (4.38),
M - Mo = n (t - tO)
M = Mo + n (t - tO)
M = 1.37113 + 0.00097202 (1,200 - 0)
M = 2.53755
METHOD #1, Low Accuracy:
Equation (4.42),
~ M + 2 e sin M + 1.25 e2 sin 2M
~ 2.53755 + 2 0.1 sin(2.53755) + 1.25 0.12 sin(2 2.53755)
~ 2.63946 = 151.2 degrees
METHOD #2, High Accuracy:
Equation (4.41),
M = E - e sin E
2.53755 = E - 0.1 sin E
By iteration, E = 2.58996 radians
Equation (4.40),
cos E = (e + cos

) / (1 + e cos

Rearranging variables gives,


cos

= (cos E - e) / (1 - e cos E)

= arccos[(cos(2.58996) - 0.1) / (1 - 0.1 cos(2.58996)]


= 2.64034 = 151.3 degrees

PROBLEM 4.15
For the satellite in problems 4.13 and 4.14, calculate the length of its position
vector, its flight-path angle, and its velocity when the satellite's true anomaly
is 225 degrees.

SOLUTION,
Given:

a = 7,500,000 m
e = 0.1
= 225 degrees

Equations (4.43) and (4.44),


r = a (1 - e2) / (1 + e cos )
r = 7,500,000 (1 - 0.12) / (1 + 0.1 cos(225))
r = 7,989,977 m
= arctan[ e sin
/ (1 + e cos )]
= arctan[ 0.1 sin(225) / (1 + 0.1 cos(225))]
= -4.351 degrees
Equation (4.45),

v = SQRT[ GM (2 / r - 1 / a)]
v = SQRT[ 3.9860051014 (2 / 7,989,977 - 1 / 7,500,000)]
v = 6,828 m/s

PROBLEM 4.16
Calculate the perturbations in longitude of the ascending node and argument of
perigee caused by the Moon and Sun for the International Space Station orbiting
at an altitude of 400 km, an inclination of 51.6 degrees, and with an orbital
period of 92.6 minutes.

SOLUTION,
Given:

i = 51.6 degrees
n = 1436 / 92.6 = 15.5 revolutions/day

Equations (4.46) through (4.49),


Moon
Moon
Moon

Sun
Sun
Sun

Moon
Moon
Moon

= -0.00338 cos(i) / n
= -0.00338 cos(51.6) / 15.5
= -0.000135 deg/day
= -0.00154 cos(i) / n
= -0.00154 cos(51.6) / 15.5
= -0.0000617 deg/day
= 0.00169 (4 - 5 sin2 i) / n
= 0.00169 (4 - 5 sin2 51.6) / 15.5
= 0.000101 deg/day

Sun
Sun
Sun

= 0.00077 (4 - 5 sin2 i) / n
= 0.00077 (4 - 5 sin2 51.6) / 15.5
= 0.000046 deg/day

PROBLEM 4.17
A satellite is in an orbit with a semi-major axis of 7,500 km, an inclination
of 28.5 degrees, and an eccentricity of 0.1. Calculate the J2 perturbations in
longitude of the ascending node and argument of perigee.

SOLUTION,
Given:

a = 7,500 km
i = 28.5 degrees
e = 0.1

Equations (4.50) and (4.51),


J2
J2
J2

J2
J2
J2

= -2.064741014 a-7/2 (cos i) (1 - e2)-2


= -2.064741014 (7,500)-7/2 (cos 28.5) (1 - (0.1)2)-2
= -5.067 deg/day
= 1.032371014 a-7/2 (4 - 5 sin2 i) (1 - e2)-2
= 1.032371014 (7,500)-7/2 (4 - 5 sin2 28.5) (1 - (0.1)2)-2
= 8.250 deg/day

PROBLEM 4.18
A satellite is in a circular Earth orbit at an altitude of 400 km. The satellite
has a cylindrical shape 2 m in diameter by 4 m long and has a mass of 1,000 kg.
The satellite is traveling with its long axis perpendicular to the velocity
vector and it's drag coefficient is 2.67. Calculate the perturbations due to
atmospheric drag and estimate the satellite's lifetime.

SOLUTION,
Given:

a = (6,378.14 + 400) 1,000 = 6,778,140 m


A = 2 4 = 8 m2
m = 1,000 kg
CD = 2.67

From Atmosphere Properties,


= 2.6210-12 kg/m3
H = 58.2 km
Equation (4.6),
V = SQRT[ GM / a ]
V = SQRT[ 3.9860051014 / 6,778,140 ]
V = 7,669 m/s
Equations (4.53) through (4.55),
arev = (-2
CD A
a2) / m
arev = (-2
2.67 8 2.6210-12 6,778,1402) / 1,000
arev = -16.2 m
2
Prev = (-6
CD A
a2) / (m V)
2
Prev = (-6
2.67 8 2.6210-12 6,778,1402) / (1,000 7,669)
Prev = -0.0199 s

Vrev = (
CD A
a V) / m
Vrev = (
2.67 8 2.6210-12 6,778,140 7,669) / 1,000
Vrev = 0.00914 m/s
Equation (4.56),
L ~ -H /
arev
L ~ -(58.2 1,000) / -16.2
L ~ 3,600 revolutions

PROBLEM 4.19
A spacecraft is in a circular parking orbit with an altitude of 200 km.
Calculate the velocity change required to perform a Hohmann transfer to a
circular orbit at geosynchronous altitude.

SOLUTION,
Given:

rA = (6,378.14 + 200) 1,000 = 6,578,140 m

From problem 4.3,


rB = 42,164,170 m
Equations (4.58) through (4.65),
atx = (rA + rB) / 2
atx = (6,578,140 + 42,164,170) / 2
atx = 24,371,155 m
ViA = SQRT[ GM / rA ]
ViA = SQRT[ 3.9860051014 / 6,578,140 ]

ViA = 7,784 m/s


VfB = SQRT[ GM / rB ]
VfB = SQRT[ 3.9860051014 / 42,164,170 ]
VfB = 3,075 m/s
VtxA = SQRT[ GM (2 / rA - 1 / atx)]
VtxA = SQRT[ 3.9860051014 (2 / 6,578,140 - 1 / 24,371,155)]
VtxA = 10,239 m/s
VtxB = SQRT[ GM (2 / rB - 1 / atx)]
VtxB = SQRT[ 3.9860051014 (2 / 42,164,170 - 1 / 24,371,155)]
VtxB = 1,597 m/s
VA = VtxA - ViA
VA = 10,239 - 7,784
VA = 2,455 m/s
VB = VfB - VtxB
VB = 3,075 - 1,597
VB = 1,478 m/s
VT =
VA +
VB
VT = 2,455 + 1,478
VT = 3,933 m/s

PROBLEM 4.20
A satellite is in a circular parking orbit with an altitude of 200 km. Using
a one-tangent burn, it is to be transferred to geosynchronous altitude using a
transfer ellipse with a semi-major axis of 30,000 km. Calculate the total
required velocity change and the time required to complete the transfer.

SOLUTION,
Given:

rA = (6,378.14 + 200) 1,000 = 6,578,140 m


rB = 42,164,170 m
atx = 30,000 1,000 = 30,000,000 m

Equations (4.66) through (4.68),


e = 1 - rA / atx
e = 1 - 6,578,140 / 30,000,000
e = 0.780729
= arccos[(atx (1 - e2) / rB - 1) / e ]
= arccos[(30,000,000 (1 - 0.7807292) / 42,164,170 - 1) / 0.780729 ]
= 157.670 degrees
= arctan[ e sin
/ (1 + e cos )]
= arctan[ 0.780729 sin(157.670) / (1 + 0.780729 cos(157.670))]
= 46.876 degrees
Equations (4.59) through (4.63),
ViA = SQRT[ GM / rA ]
ViA = SQRT[ 3.9860051014 / 6,578,140 ]
ViA = 7,784 m/s
VfB = SQRT[ GM / rB ]
VfB = SQRT[ 3.9860051014 / 42,164,170 ]
VfB = 3,075 m/s
VtxA = SQRT[ GM (2 / rA - 1 / atx)]
VtxA = SQRT[ 3.9860051014 (2 / 6,578,140 - 1 / 30,000,000)]
VtxA = 10,388 m/s
VtxB = SQRT[ GM (2 / rB - 1 / atx)]

VtxB = SQRT[ 3.9860051014 (2 / 42,164,170 - 1 / 30,000,000)]


VtxB = 2,371 m/s
VA = VtxA - ViA
VA = 10,388 - 7,784
VA = 2,604 m/s
Equation (4.69),
VB = SQRT[ VtxB2 + VfB2 - 2 VtxB VfB cos
]
VB = SQRT[ 2,3712 + 3,0752 - 2 2,371 3,075 cos(46.876)]
VB = 2,260 m/s
Equation (4.65),
VT =
VA +
VB
VT = 2,604 + 2,260
VT = 4,864 m/s
Equations (4.70) and (4.71),
E = arccos[(e + cos ) / (1 + e cos )]
E = arccos[(0.780729 + cos(157.670)) / (1 + 0.780729 cos(157.670))]
E = 2.11688 radians
TOF = (E - e sin E) SQRT[ atx3 / GM ]
TOF = (2.11688 - 0.780729 sin(2.11688)) SQRT[ 30,000,0003 / 3.9860051014 ]
TOF = 11,931 s = 3.314 hours

PROBLEM 4.21
Calculate the velocity change required to transfer a satellite from a circular

600 km orbit with an inclination of 28 degrees to an orbit of equal size with


an inclination of 20 degrees.

SOLUTION,
Given:

r = (6,378.14 + 600) 1,000 = 6,978,140 m


= 28 - 20 = 8 degrees

Equation (4.6),
Vi = SQRT[ GM / r ]
Vi = SQRT[ 3.9860051014 / 6,978,140 ]
Vi = 7,558 m/s
Equation (4.73),
V = 2 Vi sin( /2)
V = 2 7,558 sin(8/2)
V = 1,054 m/s

PROBLEM 4.22
A satellite is in a parking orbit with an altitude of 200 km and an inclination
of 28 degrees. Calculate the total velocity change required to transfer the
satellite to a zero-inclination geosynchronous orbit using a Hohmann transfer
with a combined plane change at apogee.
Given:

rA = (6,378.14 + 200) 1,000 = 6,578,140 m


rB = 42,164,170 m
= 28 degrees

From problem 4.19,

VfB = 3,075 m/s


VtxB = 1,597 m/s
VA = 2,455 m/s
Equation (4.74),
VB = SQRT[ VtxB2 + VfB2 - 2 VtxB VfB cos
]
VB = SQRT[ 1,5972 + 3,0752 - 2 1,597 3,075 cos(28)]
VB = 1,826 m/s
Equation (4.65),
VT =
VA +
VB
VT = 2,455 + 1,826
VT = 4,281 m/s

PROBLEM 4.23
A spacecraft is in an orbit with an inclination of 30 degrees and the longitude
of the ascending node is 75 degrees. Calculate the angle change required to
change the inclination to 32 degrees and the longitude of the ascending node to
80 degrees.

SOLUTION,
Given:

ii = 30 degrees
i = 75 degrees
if = 32 degrees
f = 80 degrees

Equation (4.75),
a1 = sin(ii)cos(

i) = sin(30)cos(75) = 0.129410

a2 = sin(ii)sin(

i) = sin(30)sin(75) = 0.482963

a3 = cos(ii) = cos(30) = 0.866025


b1 = sin(if)cos(

f) = sin(32)cos(80) = 0.0920195

b2 = sin(if)sin(

f) = sin(32)sin(80) = 0.521869

b3 = cos(if) = cos(32) = 0.848048


= arccos(a1 b1 + a2 b2 + a3 b3)
= arccos(0.129410 0.0920195 + 0.482963 0.521869 + 0.866025 0.848048)
= 3.259 degrees

PROBLEM 4.24
Calculate the latitude and longitude of the intersection nodes between the
initial and final orbits for the spacecraft in problem 4.23.

SOLUTION,
From problem 4.21,
a1
a2
a3
b1

=
=
=
=

0.129410
0.482963
0.866025
0.0920195

b2 = 0.521869
b3 = 0.848048
Equations (4.76) and (4.77),
c1 = a2 b3 - a3 b2 = 0.482963 0.848048 - 0.866025 0.521869 = -0.0423757
c2 = a3 b1 - a1 b3 = 0.866025 0.0920195 - 0.129410 0.848048 = -0.0300543
c3 = a1 b2 - a2 b1 = 0.129410 0.521869 - 0.482963 0.0920195 = 0.0230928
lat1 = arctan(c3 / (c12 + c22)1/2)
lat1 = arctan(0.0230928 / (-0.04237572 + -0.03005432)1/2)
lat1 = 23.965 degrees
long1 = arctan(c2 / c1) + 90
long1 = arctan(-0.0300543 / -0.0423757) + 90
long1 = 125.346 degrees
lat2 = -23.965 degrees
long2 = 125.346 + 180 = 305.346 degrees

PROBLEM 4.25
Calculate the escape velocity of a spacecraft launched from an Earth orbit with
an altitude of 200 km.

SOLUTION,
Given:

r = (6,378.14 + 200) 1,000 = 6,578,140 m

Equation (4.78),

Vesc = SQRT[ 2 GM / r ]
Vesc = SQRT[ 2 3.9860051014 / 6,578,140 ]
Vesc = 11,009 m/s

PROBLEM 4.26
A space probe is approaching Mars on a hyperbolic flyby trajectory. When at
a distance of 100,000 km, its velocity relative to Mars is 5,140.0 m/s and
its flight path angle is -85.300 degrees. Calculate the probe's eccentricity,
semi-major axis, turning angle, angle , true anomaly, impact parameter,
periapsis radius, and parameter p.

SOLUTION,
From Basics Constants,
GM of Mars = 4.2828311013 m3/s2
Given:

r = 100,000 1,000 = 100,000,000 m


v = 5,140.0 m/s
= -85.300o

Equations (4.30) and (4.32),


e = SQRT[ (r v2 / GM - 1)2 cos2
+ sin2
]
2
e = SQRT[ (100,000,000 5,140 / 4.2828311013 - 1)2 cos2(-85.3) + sin2(-85.3) ]
e = 5.0715
a = 1 / ( 2 / r - v2 / GM )
a = 1 / ( 2 / 100,000,000 - 5,1402 / 4.2828311013 )
a = -1,675,400 m

Equations (4.80) through (4.85),


sin( /2) = 1 / e
= 2 arcsin( 1 / 5.0715 )
= 22.744o
cos
= -1 / e
= arccos( -1 / 5.0715 )
= 101.37o
= arccos[ (a (1 - e2) - r) / (e r) ]
= arccos[ (-1,675,400 (1 - 5.07152) - 100,000,000) / (5.0715 100,000,000) ]
= -96.633o
b = -a / tan( /2)
b = 1,675.4 / tan(22.744/2)
b = 8,330.0 km
ro = a (1 - e)
ro = -1,675.4 (1 - 5.0715)
ro = 6,821.4 km
p = a (1 - e2)
p = -1,675.4 (1 - 5.07152)
p = 41,416 km

PROBLEM 4.27
The space probe in problem 4.26 has moved to a true anomaly of 75 degrees.
Calculate the radius vector, flight path angle, and velocity.

SOLUTION,
Given:

a = -1,675,400 m
e = 5.0715
= 75o

Equations (4.43) through (4.45),


r = a (1 - e2) / (1 + e cos )
r = -1,675,400 (1 - 5.07152) / (1 + 5.0715 cos(75))
r = 17,909,000 m
= arctan[ e sin
/ (1 + e cos )]
= arctan[ 5.0715 sin(75) / (1 + 5.0715 cos(75))]
= 64.729o
v = SQRT[ GM (2 / r - 1 / a)]
v = SQRT[ 4.2828311013 (2 / 17,909,000 - 1 / -1,675,400)]
v = 5,508.7 m/s

PROBLEM 4.28
A spacecraft is launched from Earth on a hyperbolic trajectory with a semi-major
axis of -36,000 km and an eccentricity of 1.1823. How long does it take to move
from a true anomaly of 15 degrees to a true anomaly of 120 degrees?

SOLUTION,
Given:

a = -36,000 1,000 = -36,000,000 m


e = 1.1823
o
O = 15

= 120o
Equation (4.87),
cosh F = (e + cos

) / (1 + e cos

Fo = arccosh[(1.1823 + cos(15)) / (1 + 1.1823 cos(15))]


Fo = 0.07614
F = arccosh[(1.1823 + cos(120)) / (1 + 1.1823 cos(120))]
F = 1.10023
Equation (4.86),
t - tO = SQRT[(-a)3 / GM ] [(e sinh F - F) - (e sinh Fo - Fo)]
t - tO = SQRT[(36,000,000)3 / 3.9860051014 ] [(1.1823 sinh(1.10023) - 1.10023)
- (1.1823 sinh(0.07614) - 0.07614)]
t - tO = 5,035 s = 1.399 hours

PROBLEM 4.29
A spacecraft launched from Earth has a burnout velocity of 11,500 m/s at an
altitude of 200 km. What is the hyperbolic excess velocity?

SOLUTION,
Given:

Vbo = 11,500 m/s

From problem 4.25,


Vesc = 11,009 m/s

Equation (4.88),
V
V
V

= Vbo2 - Vesc2
= SQRT[ 11,5002 - 11,0092 ]
= 3,325 m/s

PROBLEM 4.30
Calculate the radius of Earth's sphere of influence.

SOLUTION,
From Basics Constants,
Dsp = 149,597,870 km
MP = 5.97371024 kg
MS = 1.98911030 kg
Equation (4.89),
REarth = Dsp (MP / MS)0.4
REarth = 149,597,870 (5.97371024 / 1.98911030)0.4
REarth = 925,000 km

PROBLEM 5.1
Using a one-tangent burn, calculate the change in true anomaly and the

time-of-flight for a transfer from Earth to Mars. The radius vector of Earth at
departure is 1.000 AU and that of Mars at arrival is 1.524 AU. The semi-major
axis of the transfer orbit is 1.300 AU.

SOLUTION,
Given:

rA = 1.000 AU
rB = 1.524 AU
atx = 1.300 AU 149.597870109 m/AU = 194.48109 m

From Basics Constants,


GM of Sun = 1.3271241020 m3/s2
Equations (4.66) and (4.67),
e = 1 - rA / atx
e = 1 - 1.0 / 1.3
e = 0.230769
= arccos[(atx (1 - e2) / rB - 1) / e ]
= arccos[(1.3 (1 - 0.2307692) / 1.524 - 1) / 0.230769 ]
= 146.488 degrees
Equations (4.70) and (4.71),
E = arccos[(e + cos ) / (1 + e cos )]
E = arccos[(0.230769 + cos(146.488)) / (1 + 0.230769 cos(146.488))]
E = 2.41383 radians
TOF = (E - e sin E) SQRT[ atx3 / GM ]
TOF = (2.41383 - 0.230769 sin(2.41383)) SQRT[ (194.48109)3 / 1.3271241020 ]
TOF = 16,827,800 s = 194.77 days

PROBLEM 5.2
For the transfer orbit in problem 5.1, calculate the departure phase angle, given
that the angular velocity of Mars is 0.5240 degrees/day.

SOLUTION,
o
2- 1 = 146.488
t2-t1 = 194.77 days
o
t = 0.5240 /day

Given:

Equation (5.1),
= ( 2- 1) t (t2-t1)
= 146.488 - 0.5240 194.77
= 44.43o

PROBLEM 5.3
A flight to Mars is launched on 2020-7-20, 0:00 UT. The planned time of flight
is 207 days. Earth's postion vector at departure is 0.473265X - 0.899215Y AU.
Mars' postion vector at intercept is 0.066842X + 1.561256Y + 0.030948Z AU.
Calculate the parameter and semi-major axis of the transfer orbit.

SOLUTION,
Given:

t = 207 days
r1 = 0.473265X - 0.899215Y AU
r2 = 0.066842X + 1.561256Y + 0.030948Z AU

GM = 1.3271241020 m3/s2
= 1.3271241020 / (149.597870109)3 = 3.96401610-14 AU3/s2
From vector magnitude,
r1 = SQRT[ 0.4732652 + (-0.899215)2 ]
r1 = 1.016153 AU
r2 = SQRT[ 0.0668422 + 1.5612562 + 0.0309482 ]
r2 = 1.562993 AU
From vector dot product,
= arccos[ (0.473265 0.066842 - 0.899215 1.561256) / (1.016153 1.562993) ]
= 149.770967o
Equations (5.9), (5.10) and (5.11),
k = r1 r2 (1 - cos
)
k = 1.016153 1.562993 (1 - cos(149.770967))
k = 2.960511 AU
= r1 + r2
= 1.016153 + 1.562993
= 2.579146 AU
m = r1 r2 (1 + cos
)
m = 1.016153 1.562993 (1 + cos(149.770967))
m = 0.215969 AU
Equations (5.18) and (5.19),
pi = k / ( + SQRT(2 m))
pi = 2.960511 / (2.579146 + SQRT(2 0.215969))
pi = 0.914764 AU
pii = k / (

- SQRT(2 m))

pii = 2.960511 / (2.579146 - SQRT(2 0.215969))


pii = 1.540388 AU
Since

<

, 0.914764 < p <

Equation (5.12),
Select trial value, p = 1.2 AU
a = m k p / [(2 m - 2) p2 + 2 k
p - k2]
a = 0.215969 2.960511 1.2
/ [(2 0.215969 - 2.5791462) 1.22 + 2 2.960511 2.579146 1.2 - 2.9605112]
a = 1.270478 AU
Equations (5.5), (5.6) and (5.7),
f = 1 - r2 / p (1 - cos
)
f = 1 - 1.562993 / 1.2 (1 - cos(149.770967))
f = -1.427875
g = r1 r2 sin
/ SQRT[ GM p ]
g = 1.016153 1.562993 sin(149.770967) / SQRT[ 3.96401610-14 1.2 ]
g = 3,666,240
= SQRT[ GM / p ] tan(
/2) [(1 - cos
) / p - 1/r1 - 1/r2 ]
= SQRT[ 3.96401610-14 / 1.2 ] tan(149.770967/2)
[(1 - cos(149.770967)) / 1.2 - 1/1.016153 - 1/1.562993 ]
= -4.74760110-8
Equation (5.13),
E = arccos[ 1 - r1 / a (1 - f) ]
E = arccos[ 1 - 1.016153 / 1.270478 (1 + 1.427875) ]
E = 2.798925 radians
Equation (5.16),

t = g + SQRT[ a3 / GM ] ( E - sin
E)
t = 3,666,240 + SQRT[ 1.2704783 / 3.96401610-14 ] (2.798925 - sin(2.798925))
t = 21,380,951 s = 247.4647 days
Select new trial value of p and repeat above steps,
p = 1.300000 AU,

a = 1.443005 AU,

t = 178.9588 days

Equation (5.20),
pn+1 = pn + (t - tn) (pn - pn-1) / (tn - tn-1)
pn+1 = 1.3 + (207 - 178.9588) (1.3 - 1.2) / (178.9588 - 247.4647)
pn+1 = 1.259067 AU
Recalculate using new value of p,
p = 1.259067 AU,

a = 1.336197 AU,

t = 201.5624 days

Perform additional iterations,


p = 1.249221 AU,
p = 1.250673 AU,
p = 1.250633 AU,

a = 1.318624 AU,
a = 1.321039 AU,
a = 1.320971 AU,

t = 207.9408 days
t = 206.9733 days
t = 206.9999 days <-- close enough

PROBLEM 5.4
For the Mars transfer orbit in Problem 5.3, calculate the departure and intecept
velocity vectors.

SOLUTION,
Given:

r1 = 0.473265X - 0.899215Y AU
r2 = 0.066842X + 1.561256Y + 0.030948Z AU

r1 = 1.016153 AU
r2 = 1.562993 AU
p = 1.250633 AU
a = 1.320971 AU
= 149.770967o
Equations (5.5), (5.6) and (5.7),
f = 1 - r2 / p (1 - cos
)
f = 1 - 1.562993 / 1.250633 (1 - cos(149.770967))
f = -1.329580
g = r1 r2 sin
/ SQRT[ GM p ]
g = 1.016153 1.562993 sin(149.770967) / SQRT[ 3.96401610-14 1.250633 ]
g = 3,591,258
= SQRT[ GM / p ] tan(
/2) [(1 - cos
) / p - 1/r1 - 1/r2 ]
-14
= SQRT[ 3.96401610
/ 1.250633 ] tan(149.770967/2)
[(1 - cos(149.770967)) / 1.250633 - 1/1.016153 - 1/1.562993 ]
= -8.79587210-8
= 1 - r1 / p (1 - cos
)
= 1 - 1.016153 / 1.250633 (1 - cos(149.770967))
= -0.514536
Equation (5.3),
v1 = (r2 - f r1)

/ g

v1 = [(0.066842 + 1.329580 0.473265) / 3,591,258] X


+ [(1.561256 + 1.329580 -0.899215) / 3,591,258] Y
+ [(0.030948 + 1.329580 0) / 3,591,258] Z
v1 = 0.000000193828X + 0.000000101824Y + 0.00000000861759Z AU/s 149.597870109
v1 = 28996.2X + 15232.7Y + 1289.2Z m/s
Equation (5.4),

v2 =

r1 +

v1

v2 = [-8.79587210-8 0.473265 - 0.514536 0.000000193828] X


+ [-8.79587210-8 -0.899215 - 0.514536 0.000000101824] Y
+ [-8.79587210-8 0 - 0.514536 0.00000000861759] Z
v2 = -0.000000141359X + 0.0000000267017Y - 0.00000000443406Z AU/s 149.597870109
v2 = -21147.0X + 3994.5Y - 663.3Z m/s

PROBLEM 5.5
For the Mars transfer orbit in Problems 5.3 and 5.4, calculate the orbital elements.

SOLUTION,
Problem can be solved using either r1 & v1 or r2 & v2 we will use r1 & v1.
Given:

r1 =
=
r1 =
GM =

(0.473265X - 0.899215Y AU) 149.597870109 m/AU


7.0799441010X - 1.3452061011Y m
1.016153 149.597870109 = 1.5201441011 m
1.3271241020 m3/s2

From problem 5.4,


v1 = 28996.2X + 15232.7Y + 1289.2Z m/s
Also,
v = SQRT[ 28996.22 + 15232.72 + 1289.22 ] = 32,779.2 m/s
Equations (5.21) and (5.22),

h = (rY vZ - rZ vY)X + (rZ vX - rX vZ)Y + (rX vY - rY vX)Z


h = (-1.3452061011 1289.2 - 0 15232.7)X + (0 28996.2 - 7.0799441010 1289.2)Y
+ (7.0799441010 15232.7 + 1.3452061011 28996.2)Z
h = -1.734241014X - 9.127461013Y + 4.979051015Z
n = -hY X + hX Y
n = 9.127461013X - 1.734241014Y
Also,
h = SQRT[ (-1.734241014)2 + (-9.127461013)2 + (4.979051015)2 ] = 4.982911015
n = SQRT[ (9.127461013)2 + (1.734241014)2 ] = 1.959771014
Equation (5.23),
e = [(v2 - GM / r) r - (r v) v ] / GM
v2 - GM / r = 32779.22 - 1.3271241020 / 1.5201441011 = 2.01451108
r v = 7.0799441010 28996.2 - 1.3452061011 15232.7 + 0 x 1289.2 = 3.802781012
e = [2.01451108 (7.0799441010X - 1.3452061011Y)
- 3.802781012 (28996.2X + 15232.7Y + 1289.2Z) ] / 1.3271241020
e = 0.106639X - 0.204632Y - 0.000037Z
Equations (5.24) and (5.25),
a = 1 / ( 2 / r - v2 / GM )
a = 1 / ( 2 / 1.5201441011 - 32779.22 / 1.3271241020 )
a = 1.976141011 m
e = SQRT[ 0.1066392 + (-0.204632)2 + (-0.000037)2 ]
e = 0.230751
Equations (5.26) though (5.30),
cos i = hZ / h
cos i = 4.979051015 / 4.982911015

i = 2.255o
cos
= nX / n
cos
= 9.127461013 / 1.959771014
= 297.76o
cos
cos

= n e / (n e)
= (9.127461013 0.106639 - 1.734241014 (-0.204632) + 0 (-0.000037))
/ (1.959771014 0.230751)
= 359.77o

cos
cos
o

= e r / (e r)
10
- 0.204632 (-1.3452061011) - 0.000037 0)
o = (0.106639 7.07994410
11
/ (0.230751 1.52014410 )
= 0.226o
o

cos uo = n r / (n r)
uo = 0 (launch point = ascending node)
Equations (5.31) and (5.32),
=
+
= 297.76 + 359.77
= 297.53o
o
o
o

=
+
+ o
= 297.76 + 359.77 + 0.23
= 297.76o

PROBLEM 5.6

For the spacecraft in Problems 5.3 and 5.4, calculate the hyperbolic excess
velocity at departure, the injection
V, and the zenith angle of the departure
asymptote. Injection occurs from an 200 km parking orbit. Earth's velocity
vector at departure is 25876.6X + 13759.5Y m/s.

SOLUTION,
Given:

ro = (6,378.14 + 200) 1,000 = 6,578,140 m


r = 0.473265X - 0.899215Y AU
VP = 25876.6X + 13759.5Y m/s

From problem 5.4,


VS = 28996.2X + 15232.7Y + 1289.2Z m/s
Equation (5.33),
VS/P = (VSX - VPX)X + (VSY - VPY)Y + (VSZ - VPZ)Z
VS/P = (28996.2 - 25876.6)X + (15232.7 - 13759.5)Y + (1289.2 - 0)Z
VS/P = 3119.6X + 1473.2Y + 1289.2Z m/s
Equation (5.34),
VS/P = SQRT[ VS/PX2 + VS/PY2 + VS/PZ2 ]
VS/P = SQRT[ 3119.62 + 1473.22 + 1289.22 ]
VS/P = 3,683.0 m/s
V

= VS/P = 3,683.0 m/s

Equations (5.35) and (5.36),


Vo = SQRT[ V 2 + 2 GM / ro ]
Vo = SQRT[ 3,683.02 + 2 3.9860051014 / 6,578,140 ]
Vo = 11,608.4 m/s
V = Vo - SQRT[ GM / ro ]
V = 11,608.4 - SQRT[ 3.9860051014 / 6,578,140 ]

V = 3,824.1 m/s
Equation (5.37),
r = SQRT[ 0.4732652 + (-0.899215)2 + 02 ]
r = 1.01615 AU
= arccos[(rX vX + rY vY + rZ vZ) / (r v)]
= arccos[ 0.473265 3119.6 - 0.899215 1473.2 + 0 1289.2) / (1.01615 3683.0)]
= 87.677o

PROBLEM 5.7
For the spacecraft in Problems 5.3 and 5.4, given a miss distance of +18,500 km
at arrival, calculate the hyperbolic excess velocity, impact parameter, and
semi-major axis and eccentricity of the hyperbolic approach trajectory. Mars'
velocity vector at intercept is -23307.8X + 3112.0Y + 41.8Z m/s.

SOLUTION,
Given:

d = 18,500 km / 149.597870106 = 0.000123664 AU


r = 0.066842X + 1.561256Y + 0.030948Z AU
VP = -23307.8X + 3112.0Y + 41.8Z m/s

From Basics Constants,


GM of Mars = 4.2828311013 m3/s2
From problem 5.4,
VS = -21147.0X + 3994.5Y - 663.3Z m/s

Equation (5.33),
VS/P = (VSX - VPX)X + (VSY - VPY)Y + (VSZ - VPZ)Z
VS/P = (-21147.0 + 23307.8)X + (3994.5 - 3112.0)Y + (-663.3 - 41.8)Z
VS/P = 2160.8X + 882.5Y - 705.1Z m/s
Equation (5.34),
VS/P = SQRT[ VS/PX2 + VS/PY2 + VS/PZ2 ]
VS/P = SQRT[ 2160.82 + 882.52 + (-705.1)2 ]
VS/P = 2,438.2 m/s
V

= VS/P = 2,438.2 m/s

Equations (5.38.A) and (5.38.B),


dx = -d ry / SQRT[ rx2 + ry2 ]
dx = -0.000123664 1.561256 / SQRT[ 0.0668422 + 1.5612562 ]
dx = -0.000123551 AU
dy = d rx / SQRT[ rx2 + ry2 ]
dy = 0.000123664 0.066842 / SQRT[ 0.0668422 + 1.5612562 ]
dy = 0.0000052896 AU
Equation (5.39),
= arccos[(dx vx + dy vy) / (d v)]
= arccos[(-0.000123551 2160.8 + 0.0000052896 882.5) / (0.000123664 2,438.2)]
= 150.451o
Equations (5.40) through (5.42),
b = d sin
b = 18,500 sin(150.451)
b = 9,123.6 km
a = -GM / V

a = -4.2828311013 / 2,438.22
a = -7.2043106 m = -7,204.3 km
e = SQRT[ 1 + b2 / a2 ]
e = SQRT[ 1 + 9,123.62 / -7,204.32 ]
e = 1.6136

PROBLEM 5.8
As a spacecraft approaches Jupiter, it has a velocity of 9,470 m/s, a flight
path angle of 39.2 degrees, and a targeted miss distance of -2,500,000 km. At
intercept, Jupiter's velocity is 12,740 m/s with a flight path angle of 2.40
degrees. Calculate the spacecraft's velocity and flight path angle following
its swing-by of Jupiter.
Given:

VP = 12,740 m/s
o
P = 2.40
VSi = 9,470 m/s
Si = 39.2o
d = -2,500,000 km

From Basics Constants,


GM of Jupiter = 1.266861017 m3/s2
Equations (5.44) and (5.45),
VP = (VP cos P)X + (VP sin P)Y
VP = (12740 cos(2.40))X + (12740 sin(2.40))Y
VP = 12729X + 533Y m/s
VSi = (VSi cos Si)X + (VSi sin Si)Y
VSi = (9470 cos(39.2))X + (9470 sin(39.2))Y

VSi = 7339X + 5985Y m/s


Equations (5.46) and (5.47),
VS/Pi = ((VSi)X - VPX)X + ((VSi)Y - VPY)Y
VS/Pi = (7339 - 12729)X + (5985 - 533)Y
VS/Pi = -5390X + 5452Y m/s
VS/P = SQRT[ (VS/Pi)X2 + (VS/Pi)Y2 ]
VS/P = SQRT[ (-5390)2 + 54522 ]
VS/P = 7,667 m/s
V

= VS/P = 7,667 m/s

Equation (5.48),
i
i
i

= arctan[ (VS/Pi)Y / (VS/Pi)X ]


= arctan[ 5452 / -5390 ]
= 134.67o

Equations (5.40) through (5.42),


b = d sin
b = -2,500,000 sin(134.67)
b = -1,777,900 km
a = -GM / V 2
a = -1.266861017 / 76672
a = -2.1552109 m = -2,155,200 km
e = SQRT[ 1 + b2 / a2 ]
e = SQRT[ 1 + (-1,777,900)2 / (-2,155,200)2 ]
e = 1.2963
Equation (5.49),
= 2 arcsin( 1 / e )

= 2 arcsin( 1 / 1.2963 )
= 100.96o
Equation (5.50),
f
f
f

= i = 134.67 - 100.96
= 33.71o

Equation (5.51),
VS/Pf = (VS/P cos f)X + (VS/P sin f)Y
VS/Pf = (7667 cos(33.71))X + (7667 sin(33.71))Y
VS/Pf = 6378X + 4255Y m/s
Equations (5.52) and (5.53),
VSf = ((VS/Pf)X + VPX)X + ((VS/Pf)Y + VPY)Y
VSf = (6378 + 12729)X + (4255 + 533)Y
VSf = 19107X + 4788Y m/s
VSf = SQRT[ (VSf)X2 + (VSf)Y2 ]
VSf = SQRT[ 191072 + 47882 ]
VSf = 19,698 m/s
Equation (5.54),
Sf
Sf
Sf

= arctan[ (VSf)Y / (VSf)X ]


= arctan[ 4788 / 19107 ]
= 14.07o

Vous aimerez peut-être aussi